11th may 2019 live test conducted by

102
11 th May 2019 Live Test conducted by Vajiram & Ravi Based on March 2019 & April 2019 current affairs Q1. With reference to the Modern Indian languages, which of the following languages of southern and central India belong to the Dravidian language family? (1) Brahui (2) Gondi (3) Malto (4) Tulu Select the correct answer using the code given below: A. 1 and 2 only B. 3 and 4 only C. 1, 2 and 4 only D. 1, 2, 3 and 4 Answer: D Explanation: The Dravidian language family’s four largest languages — Kannada, Malayalam, Tamil and Telugu — have literary traditions spanning centuries, of which Tamil reaches back the furthest. Kurukh is a Dravidian language spoken by nearly two million Oraon and Kisan tribespeople of Jharkhand, Madhya Pradesh, Chhattisgarh, Odisha and West Bengal, as well as in northern Bangladesh and in Nepal. Brahui is a Dravidian language spoken primarily by the Brahui people in the central part of Baluchistan province in Pakistan, and in scattered parts of Afghanistan, Iran, and Turkmenistan, and by expatriate Brahui communities in Qatar, United Arab Emirates, and Iraq. The Malto or Maler people are a Dravidian-speaking ethnic group. While most Dravidians are found in South India, the Malto mostly live in the northeast of South Asian subcontinent. Tulu is a Dravidian language spoken mainly in the south west part of the Indian state of Karnataka and also in the Kasaragod district of Kerala. The Tulu speaking region is often referred to as Tulu Nadu.

Upload: others

Post on 07-Nov-2021

1 views

Category:

Documents


0 download

TRANSCRIPT

Page 1: 11th May 2019 Live Test conducted by

11th May 2019 Live Test conducted by

Vajiram & Ravi

Based on March 2019 & April 2019 current affairs

Q1. With reference to the Modern Indian languages, which of the following languages of

southern and central India belong to the Dravidian language family?

(1) Brahui

(2) Gondi

(3) Malto

(4) Tulu

Select the correct answer using the code given below:

A. 1 and 2 only

B. 3 and 4 only

C. 1, 2 and 4 only

D. 1, 2, 3 and 4

Answer: D

Explanation:

The Dravidian language family’s four largest languages — Kannada, Malayalam, Tamil and

Telugu — have literary traditions spanning centuries, of which Tamil reaches back the furthest.

Kurukh is a Dravidian language spoken by nearly two million Oraon and Kisan tribespeople of

Jharkhand, Madhya Pradesh, Chhattisgarh, Odisha and West Bengal, as well as in northern

Bangladesh and in Nepal. Brahui is a Dravidian language spoken primarily by the Brahui people

in the central part of Baluchistan province in Pakistan, and in scattered parts of Afghanistan,

Iran, and Turkmenistan, and by expatriate Brahui communities in Qatar, United Arab Emirates,

and Iraq. The Malto or Maler people are a Dravidian-speaking ethnic group. While most

Dravidians are found in South India, the Malto mostly live in the northeast of South Asian

subcontinent. Tulu is a Dravidian language spoken mainly in the south west part of the Indian

state of Karnataka and also in the Kasaragod district of Kerala. The Tulu speaking region is often

referred to as Tulu Nadu.

Page 2: 11th May 2019 Live Test conducted by

Madhya Pradesh Chief Minister has recently decided to include ancient tribal language ‘GONDI’

in the primary education curriculum of the state’s tribal-dominated districts.

According to 2011 Census, Gonds, believed to belong to the Dravidian stock, have been notified

as a scheduled tribe in the states of Andhra Pradesh, Bihar, Chhattisgarh, Gujarat, Jharkhand,

Karnataka, Madhya Pradesh, Maharashtra, Odisha, Uttar Pradesh, Uttaranchal and West

Bengal. Gondi is a South-Central Dravidian language.The Gondi script is perhaps the only script

in the country besides Urdu which is written right to left. Hence, option D. is the correct

answer.

Source:http://vajiramias.com/current-affairs/gondi-language/5c80ce1c1d5def14e297cb1c/

Q2. Which of the following islands are located in the Indian Ocean?

(1) Reunion Island

(2) Assumption Island

(3) Easter Island

(4) Diego Garcia Island

Select the correct answer using the code given below:

A. 1 and 2 only

B. 2 and 4 only

C. 1, 2 and 4 only

Page 3: 11th May 2019 Live Test conducted by

D. 1, 2, 3 and 4

Answer: C

Explanation:

National space agency Indian Space Research Organisation (ISRO) and its French counterpart

National Centre for Space Studies (CNES) sealed an agreement to set up a joint maritime

surveillance system in the country. ISRO and CNES will explore putting up a constellation of low-

Earth orbiting satellites that will identify and track movement of ships globally esp. in the Indian

Ocean where France has its Reunion Islands.

Point (1) is correct: Reunion Island is an overseas department and region of France and an

island in the Indian Ocean, east of Madagascar.

Point (2) is correct: Assumption Island is a small island in the Outer Islands of Seychelles north

of Madagascar.

Point (3) is incorrect: Easter Island is a Chilean island in the south eastern Pacific Ocean, at the

south easternmost point of the Polynesian Triangle in Oceania.

Point (4) is correct: Diego García is an atoll just south of the equator in the central Indian

Ocean, and the largest of 60 small islands comprising the Chagos Archipelago.

Source: http://vajiramias.com/current-affairs/isro-cnes-agreement-on-maritime-

security/5c80c5aa1d5def14e5669c0e/

Q3. Consider the following pairs:

Page 4: 11th May 2019 Live Test conducted by

Which of the pairs given above is/are correctly matched?

A. 1 only

B. 2 and 3 only

C. 1 and 3 only

D. 1, 2 and 3

Answer: A

Explanation:

Pair (1) is correctly matched: Kalaripayattu is an Indian martial art and fighting system that

originated in Kerala.

Pair (2) is not correctly matched: Thang Ta is popular term for the ancient Manipuri Martial Art

known as Huyen Lallong.

Pair (3) is not correctly matched: Kudo is an official Japanese Cultural Budo sport under the

same category as Judo, Aikido and Kendo. Recently, Union Ministry of Youth Affairs and Sports

has granted provisional recognition to Kudo International Federation India (KIFI) as National

Sport Federation with immediate effect. KUDO is Mixed Martial Arts (MMA). It is a safe,

scientific, modern, & practical Combination of traditional & sports Karate, Judo, Jujutsu, Muay

Thai, & Kickboxing. The recognition means granting a major role to KIFI Association for

promotion and Development of Kudo sport in India. The Kudo sport is placed in the ‘Others’

category.

Source: http://vajiramias.com/current-affairs/kudo/5c80c4191d5def14e297c9c5/

Q4. With reference to the AGNIi mission, consider the following statements:

(1) It is an initiative that aims to support the ongoing efforts to boost the innovation ecosystem

in the country.

Page 5: 11th May 2019 Live Test conducted by

(2) The mission is under the Union Ministry of Defence.

Which of the statements given above is/are correct?

A. 1 only

B. 2 only

C. Both 1 and 2

D. Neither 1 nor 2

Answer: A

Explanation:

Statement (1) is correct: Accelerating Growth of New India’s Innovations or AGNIi is a national

initiative of the Government of India that aims to support the ongoing efforts to boost the

innovation ecosystem in the country by connecting innovators across industry, individuals and

the grassroots to the market and helping commercialise their innovative solutions. It will

provide a platform for innovators to bring their market ready products and solutions to industry

thereby helping propel India into a new era of inclusive economic growth.

Statement (2) is incorrect: It is an initiative under the guidance and support of the Principal

Scientific Adviser to the Government of India. The Prime Minister’s Science, Technology and

Innovation Advisory Council (PM-STIAC) has recently launched nine mission of national

importance, including those on artificial intelligence, quantum computing, electric vehicles and

deep ocean exploration.

Source: http://vajiramias.com/current-affairs/nine-science-and-technology-

missions/5c80c7ca1d5def14e5669c6c/

Q5. With respect to the “Pradhan Mantri Bhartiya Janaushadhi Pariyojana (PMBJP)”, consider

the following statements:

(1) Its objective is to make available quality generic medicines at affordable prices to all.

(2) It is implemented by the Department of Pharmaceuticals under the Ministry of Ayush.

Which of the statements given above is/are correct?

A. 1 only

B. 2 only

Page 6: 11th May 2019 Live Test conducted by

C. Both 1 and 2

D. Neither 1 nor 2

Answer: A

Explanation:

Statement (1) is correct: Pradhan Mantri Bhartiya Janaushadhi Pariyojana (PMBJP)’s objective is

to make available quality generic medicines at affordable prices to all.

Statement (2) is incorrect: It is implemented by Bureau of Pharma PSUs of India (BPPI), under

the administrative control of the Department of Pharmaceuticals, Ministry of Chemicals &

Fertilizers, Government of India.

Jan Aushadhi Scheme was launched by the Department of Pharmaceuticals in 2008 across the

country. In 2016, it was again renamed as Pradhan Mantri Bhartiya Janaushadhi Pariyojana

(PMBJP).7th March 2019 was celebrated as ‘Janaushadhi Diwas’ across India for creating

awareness about use of generic medicines.

Source:

http://vajiramias.com/current-affairs/janaushadhi-diwas/5c8235671d5def6ca21eaf0b/

Q6. Consider the following statements:

(1) The Nice Agreement is for setting up an International classification of the figurative

elements of marks.

(2) The Locarno Agreement is for establishing an International classification for industrial

designs.

(3) The Vienna Agreement is on the International classification of goods and services for the

purposes of registration of marks.

Which of the statements given above is/are correct?

A. 2 only

B. 2 and 3 only

C. 1 and 3 only

D. 1, 2 and 3

Page 7: 11th May 2019 Live Test conducted by

Answer: A

Explanation:

The Union Cabinet has approved the proposal for accession of India to the Nice, Vienna and

Locarno Agreements.

Statement (1) is incorrect: The Nice Agreement is on the International classification of goods

and services for the purposes of registration of marks.

Statement (2) is correct: The Locarno Agreement is for establishing an International

classification for industrial designs.

Statement (3) is incorrect: The Vienna Agreement is for setting up an International classification

of the figurative elements of marks.

Accession to the Agreements will help the Intellectual Property Office (IPO) in India to

harmonise the classification systems for examinational of trademark and design applications, in

line with the classification systems followed globally. It will also give an opportunity to include

Indian designs, figurative elements and goods in the international classification systems and

instill confidence in foreign investors in relation to protection of IPs in India. It will also facilitate

in exercising rights in decision making processes regarding review and revision of the

classifications under the agreement.

Source: http://vajiramias.com/current-affairs/nice-vienna-and-locarno-

agreements/5c89e82c1d5def4480e4f707/

Q7. In which of the following Union Territories (UTs) the President of India does not have the

power to make regulations under Article 240 of the Constitution?

A. Andaman and Nicobar Islands

B. Pondicherry

C. Dadra and Nagar Haveli

D. Chandigarh

Answer: D

Explanation:

Page 8: 11th May 2019 Live Test conducted by

According to Article 240of the Constitution, the President of India has the power to make

regulations for certain UTs. UTs covered: (1) the Andaman and Nicobar Islands (2) Lakshadweep

(3) Dadra and Nagar Haveli (4) Daman and Diu and (5) Pondicherry. The regulations could be for

peace, progress and good government of the UT. The regulation made by the President may

“repeal or amend any Act made by Parliament or any other law which is for the time being

applicable to the UT.” When the regulation is announced by the President, it has the same

effect as an Act of Parliament.

Recently, the Union Cabinet has approved the Promulgation of: i) The Daman and Diu Civil

Courts (Amendment) Regulation, 2019 ii) The Dadra and Nagar Haveli (Civil Courts and

Miscellaneous Provisions) Amendment Regulation, 2019 under Article 240 of the

Constitution. The president of India, in exercise of his powers under clause (1) of article 239 of

the Constitution of India, appointed the administrator of union territory of Chandigarh to

exercise the powers and discharge those functions of the central government. Hence, option D.

is the correct answer.

Source: http://vajiramias.com/current-affairs/article-240/5c89e90a1d5def447ea82ef0/

Q8. The Global Environment Outlook (GEO) is published by which among the following

agencies?

A. United Nations Conference on Environment and Development (UNCED)

B. World Wide Fund (WWF)

C. United Nations Environmental Programme (UNEP).

D. World Watch Institute

Answer: C

Explanation:

The Global Environment Outlook (GEO) is a series of reports on the environment

issued periodically by the United Nations Environmental Programme (UNEP). The first

publication was in 1997. The recent GEO — a report six years in the making has been compiled

by 250 scientists from 70 nations. Hence, option C. is the correct answer.

Poor environmental conditions “cause approximately 25% of global disease and mortality”

around 9 million deaths in 2015 alone. Air pollution causes 6-7 million early deaths annually.

Chemicals pumped into the seas cause “potentially multi-generational” adverse health effects.

Due to Lack of access to clean drinking supplies, 1.4 million people die each year from

Page 9: 11th May 2019 Live Test conducted by

preventable diseases such as diarrhoea and parasites. Land degradation through mega-farming

and deforestation occurs in areas of Earth home to 3.2 billion people.

Source: http://vajiramias.com/current-affairs/global-environment-outlook-geo-

6/5c8a01731d5def4480e4fb72/

Q9. With reference to “FOREX SWAPS”, consider the following statements:

(1) It is an instrument to reduce rupee liquidity through long-term foreign exchange buy/sell

swap into the system.

(2) It is a contract in which one party borrows one currency from, and simultaneously lends

another to the second party.

Which of the statements given above is/are correct?

A. 1 only

B. 2 only

C. Both 1 and 2

D. Neither 1 nor 2

Answer: B

Explanation:

Statement (1) is incorrect: The Reserve Bank of India (RBI) has decided to inject rupee liquidity

into the system through long-term foreign exchange buy/sell swap — a first-of-its-kind

instrument used for liquidity management. Forex swaps have been employed to raise foreign

currencies, both for financial institutions and their customers, including exporters and

importers, as well as institutional investors who wish to hedge their positions. RBI has been an

active participant in the dollar/rupee swap market but this is the first time the central bank has

announced an auction.

Statement (2) is correct: A Forex swap agreement is a contract in which one party borrows one

currency from, and simultaneously lends another to, the second party. Each party uses the

repayment obligation to its counterparty as collateral and the amount of repayment is fixed at

the Forex forward rate as of the start of the contract.

Source: http://vajiramias.com/current-affairs/forex-swaps/5c8a0c7a1d5def4480e4fdfa/

Page 10: 11th May 2019 Live Test conducted by

Q10. With reference to the human eyes, which of the following is/are the primary type(s) of

refractive errors?

(1) Myopia

(2) Hyperopia

(3) Astigmatism

Select the correct answer using the code given below:

A. 1 only

B. 2 and 3 only

C. 1 and 3 only

D. 1, 2 and 3

Answer: D

Explanation:

The bending and focusing of light is also known as refraction. Usually the shape of the cornea

and the eye are not perfect and the image on the retina is out- of-focus (blurred) or distorted.

These imperfections in the focusing power of the eye are called refractive errors.

There are three primary types of refractive errors, they are myopia, hyperopia and astigmatism.

Persons with myopia, or nearsightedness, have more difficulty seeing distant objects as clearly

as near objects. Persons with hyperopia, or farsightedness, have more difficulty seeing near

objects as clearly as distant objects. Astigmatism is a distortion of the image on the retina

caused by irregularities in the cornea or lens of the eye. Combinations of myopia and

astigmatism or hyperopia and astigmatism are common. Hence option D. is the correct answer.

Source: http://vajiramias.com/current-affairs/lasik/5c8cac361d5def612183261f/

Q11. With reference to the Fundamental Rights, which of the following rights have been

declared as part of Article 21 by the Supreme Court?

(1) Right to self-defence

(2) Right to shelter

(3) Right to reputation

Page 11: 11th May 2019 Live Test conducted by

(4) Right to freedom from noise pollution

Select the correct answer using the code given below:

A. 1, 2 and 3 only

B. 1, 2 and 4 only

C. 2, 3 and 4 only

D. 1, 2, 3 and 4

Answer: C

Explanation:

The Supreme Court ruled that Right to Self-Defence extends not only to one’s own body but

also to protecting the person and property of another. The right also embraces the protection

of property, whether one’s own or another person’s, against certain specified offences, namely,

theft, robbery, mischief and criminal trespass”. Indian Penal Code (IPC) Section 96 to 106 of the

penal code states the law relating to the right of private defence of person and property.

Section 97 says that the right of private defence is of 2 types: (i) Right of private defence of

body, (ii) Right of private defence of property.

Article 21 declares that no person shall be deprived of his life or personal liberty except

according to procedure established by law. This right is available to both citizens and non-

citizens. The Supreme Court has declared the following rights as part of Article 21: Right to

shelter, Right to reputation, Right to freedom from noise pollution, Right to sleep Right against

bonded labour, Right against custodial harassment, Right to emergency medical aid etc. Hence,

option C. is the correct answer.

Source: http://vajiramias.com/current-affairs/right-to-self-

defence/5c91f5181d5def05bf428bea/

Q12. Which of the following pairs is/are correctly matched?

Page 12: 11th May 2019 Live Test conducted by

Select the correct answer using the code given below:

A. 1 only

B. 2 and 3 only

C. 1 and 3 only

D. 1, 2 and 3

Answer: D

Explanation:

According to the latest annual breakdown of patent filings released by the World Intellectual

Property Organization (WIPO), more than half of all international patent applications filed last

year came from Asia. The World Intellectual Property Organization is one of the 15 specialized

agencies of the United Nations. WIPO was created in 1967"to encourage creative activity, to

promote the protection of intellectual property throughout the world".

Pair (1) is correctly matched: A patent safeguards an original invention for a certain period of

time. It is granted by the Government to a company/individual for an invention.

Pair (2) is correctly matched: Unlike patents, a trademark protects words and design elements

that identify the source of a product. Brand names and corporate logos are primary examples.

A service mark is similar, except that it safeguards the provider of a service instead of a tangible

good. The term “trademark” is often used in reference to both designations.

Pair (3) is correctly matched: Copyright law primarily protects original expression in literary,

dramatic, musical and artistic “works”. For as long as the copyright is in effect, the copyright

owner has the sole right to display, share, perform or license the material. One notable

exception is the “fair use” doctrine, which allows some degree of distribution of copyrighted

Page 13: 11th May 2019 Live Test conducted by

material for scholarly, educational or news-reporting purposes. Hence, option D. is the correct

answer.

Source: http://vajiramias.com/current-affairs/patents/5c91d88c1d5def1394cd1c9a/

Q13. With which of the following Central Asian Country, Kazakhstan doesn’t share its border?

A. Turkmenistan

B. Kyrgyzstan

C. Tajikistan

D. Uzbekistan

Answer: C

Explanation:

Kazakhstan is located in Central Asia. The country borders Turkmenistan, Uzbekistan, and

Kyrgyzstan to the south (It doesn’t touches Tajikistan); Russia to the north; Russia and the

Caspian Sea to the west; and China’s Xinjiang Uygur Autonomous Region to the east. Astana is

the capital city of Kazakhstan.

Source:http://vajiramias.com/current-affairs/nursultan/5c9323c81d5def1394cd644b/

http://vajiramias.com/current-affairs/nursultan-nazarbayev/5c91d4f31d5def05bf4282f1/

Q14. According to a recent report jointly released by the World Bank and the International

Labour Organization (ILO), increasing exports can lead to better jobs, higher wages in India.

Page 14: 11th May 2019 Live Test conducted by

In this context which of the following initiatives are taken exclusively to promote exports from

the country?

(1) Special Economic Zones

(2) National Investment and Manufacturing Zones

(3) Trade Infrastructure for Export Scheme

(4) Niryat Bandhu Scheme

Select the correct answer using the code given below:

A. 1 and 3 only

B. 1, 2 and 3 only

C. 1, 3 and 4 only

D. 1, 2, 3 and 4

Answer: C

Explanation:

The Union Cabinet has approved promulgation of an Ordinance to amend the definition of

‘person’ as defined in sub-section (v) of section 2 of the Special Economic Zones Act, 2005 (28

of 2005) to include a trust, to enable the setting up of a unit in a SEZ by a trust. The present

provision of the SEZs Act, 2005 do not permit ‘trusts’ to set up units in SEZs.

Point (1) is correct: SEZs are major export hubs in the country as the government provides

several incentives including tax benefits and single-window clearance system.

Point (2) is not correct: SEZ’s objective is to promote exports, while NIMZs are based on the

principle of industrial growth in partnership with States and focuses on manufacturing growth

and employment generation. NIMZs are different from SEZs in terms of size, level of

infrastructure planning, governance structures related to regulatory procedures, and exit

policies.

Point (3) is correct: The Union Commerce Ministry launched the Trade Infrastructure for Export

Scheme (TIES) to create appropriate infrastructure for development and growth of exports

through engagement of central or state agencies.

Point (4) is correct: The objective of the Niryat Bandhu Scheme is to reach out to the potential

exporters and mentor them through orientation programmes, counselling sessions and

Page 15: 11th May 2019 Live Test conducted by

individual facilitation for being able to get into international trade and boost exports from

India. Hence, option C. is the correct answer.

Source:http://vajiramias.com/current-affairs/special-economic-

zones/5c78eb4a1d5def14e296a1e3/

Q15. Consider the following statements with reference to the TRIFOOD project:

(1) It is a joint initiative of Ministry of Food Processing Industry, Ministry of Tribal Affairs and

Food Safety and Standards Authority of India (FSSAI)

(2) The traditional Mahua tribal drink will be mainstreamed and marketed all over the Country

under this project.

Which of the statements given above is/are correct?

A. 1 only

B. 2 only

C. Both 1 and 2

D. Neither 1 nor 2

Answer: B

Explanation:

Statement (1) is incorrect: TRIFOOD Scheme is a joint initiative of Ministry of Food Processing

Industry, Ministry of Tribal Affairs and TRIFED.

Statement (2) is correct: Under this scheme a tertiary value addition center will be set up in

Jagdalpur in Chhattisgarh and Raigad in Maharashtra at a cost of approximately Rs.11 crores. A

highlight of this is the production of “Heritage Mahua” drink. The traditional Mahua tribal drink

will be mainstreamed and marketed all over the Country under this project.

Recently Ministry of Tribal Affairs launched various schemes for development of Tribals such as

Minimum Support Price for Minor Forest produces to cover 50 items, Van Dhan, Trifood and

Friends of Tribes schemes.

Source: http://vajiramias.com/current-affairs/tribal-development-

schemes/5c78f95b1d5def14e56588bd/

Q16. With reference to ‘Faster Adoption and Manufacturing of Electric Vehicles (FAME) INDIA

PHASE II’, consider the following statements:

Page 16: 11th May 2019 Live Test conducted by

(1) The benefits of incentives will be extended to only those vehicles which are fitted with

advance battery like a Lithium Ion battery and other new technology batteries.

(2) It is implemented under the aegis of Ministry of New and Renewable Energy.

Which of the statements given above is/are correct?

A. 1 only

B. 2 only

C. Both 1 and 2

D. Neither 1 nor 2

Answer: A

Explanation:

The Union cabinet has recently approved the proposal for implementation of scheme titled

‘FAME India Phase II’ for promotion of Electric Mobility in the country.

Statement (2) is incorrect: Faster Adoption and Manufacturing of Electric Vehicles in India

Phase II’s (FAME India Phase II) objective is to promote Electric and hybrid vehicle in the

country by offering upfront Incentive on purchase of Electric vehicles and by establishing

charging Infrastructure for electric vehicles. It is implemented by the Department of Heavy

Industry.

Statement (1) is correct: The scheme with total outlay of Rs 10,000 Crores over the period of

three years (2019- 20 to 2021-22) will be implemented with effect from 1st April 2019.Salient

features of the scheme: Emphasis is on electrification of the public transportation that includes

shared transport. The benefits of incentives will be extended to only those vehicles which are

fitted with advance battery like a Lithium Ion battery and other new technology batteries.

Source: http://vajiramias.com/current-affairs/fame-india-phase-

ii/5c78d0601d5def14e56583b0/

Q17. To achieve greater inflow of private capital, in which of the following organizations the

Government of India has taken the decision for strategic disinvestment?

(1) Bharat Aluminium Company Ltd. (BALCO)

(2) Indian Petrochemicals Corporation Limited (IPCL)

(3)Air India

Page 17: 11th May 2019 Live Test conducted by

Select the correct answer using the code given below:

A. 1 and 2 only

B. 2 and 3 only

C. 1 and 3 only

D. None of the above

Answer: A

Explanation:

Recently the government gave a post-facto approval for the creation of a special purpose

vehicle (SPV) for the disinvestment of Air India and its subsidiaries and joint-venture companies.

A majority disinvestment is one in which the government, post disinvestment, retains a

minority stake in the company i.e. it sells off a majority stake. It is also called Strategic

Disinvestment. In the current proposal, the government is offering 76% in Air India, which

includes 100% in low-cost international subsidiary Air India Express and 50% in a ground

handling company AISATS.

During the time period (1999-2004) Government made four strategic disinvestments in Bharat

Aluminium Company (BALCO) and Hindustan Zinc (both to Sterlite Industries), Indian

Petrochemicals Corporation Limited (to Reliance Industries) and VSNL (to the Tata group) each

of which is now managed by the private sector shareholder. Hence, option A. is the

correct answer.

Source: http://vajiramias.com/current-affairs/disinvestment/5c9612591d5def2fdbeadd84/

http://vajiramias.com/current-affairs/air-india-assets-holding-ltd-

aiahl/5c7a77eb1d5def14e565b9da/

Q18. Consider the following statements with reference to the Moon's 'sunburns':

(1) It is a distinctive pattern of swirls which is a result of interactions between the Sun's

damaging radiations with pockets of lunar magnetic field.

(2) Like Earth, the Moon also has global magnetic field.

Which of the statements given above is/are correct?

A. 1 only

Page 18: 11th May 2019 Live Test conducted by

B. 2 only

C. Both 1 and 2

D. Neither 1 nor 2

Answer: A

Explanation:

Statement (1) is correct: Research using data from NASA's Acceleration, Reconnection,

Turbulence and Electrodynamics of the Moon's Interaction with the Sun (ARTEMIS) mission

suggests that lunar swirls could be the result of solar wind interactions with the Moon's isolated

pockets of magnetic field. It is a distinctive pattern of swirls which is a result of interactions

between the Sun's damaging radiations with pockets of lunar magnetic field.

Statement (2) is incorrect: Unlike Earth, the Moon has no global magnetic field. However,

magnetised rocks near the lunar surface do create small, localised spots of magnetic field that

extend anywhere from hundreds of yards to hundreds of miles. The lunar regolith is the layer

of loose soil that covers the surface of the Moon. The regolith consists of unconsolidated

debris: dust, soil, fragments of the bedrock beneath and, as a result, is non uniform in texture.

Source: http://vajiramias.com/current-affairs/moons-

sunburns/5c7a75c81d5def14e14725b1/

Q19. According to the Census 2011, there are 2.68 Crore Persons with Disabilities (PwDs) in

India constituting 2.21% of the total population. In this context, which of the following

initiatives have been taken by the Government of India to serve the differently-abled

community of the country?

(1) The Rights of Persons with Disabilities Act, 2016

(2) National Handicapped Finance and Development Corporation (NHFDC)

(3) Sugamya Bharat Abhiyan

(4) Ratification of United Nations Convention on the Rights of Persons with

Disabilities (UNCRPD)

Select the correct answer using the code given below:

A. 1 and 2 only

B. 1, 2 and 3 only

Page 19: 11th May 2019 Live Test conducted by

C. 1, 2 and 4 only

D. 1, 2, 3 and 4

Answer: D

Explanation:

Point (1) is correct: The Union Cabinet has approved the proposal for Creation of two posts of

Commissioners in the Office of the Chief Commissioner for Persons with Disabilities (CCPD). The

recent decision has been taken in terms of Section 74(2) of The Rights of Persons with

Disabilities (RPwD) Act, 2016. Out of the two posts of Commissioners, one Commissioner will be

a person with disability.

Point (2) is correct: National Handicapped Finance and Development Corporation (NHFDC) was

set up by the Department of Disability Affairs in 1997 with the objective of serving as a catalyst

in the economic development of Persons with Disabilities (PwDs) by helping them in setting up

self-employment projects.

Point (3) is correct: Accessible India Campaign or Sugamya Bharat Abhiyan is a program which is

set to be launched to serve the differently-able community of the country.

Point (4) is correct: India signed the United Nations Convention on the Rights of Persons with

Disabilities (UNCRPD) and subsequently ratified the same on 1st October, 2007.The Rights of

Persons with Disabilities Act, 2016 replaces the Persons with Disabilities (Equal Opportunities,

Protection of Rights and Full Participation) Act, 1995 and is in line with the principles of the

United Nations Convention on the Rights of Persons with Disabilities.

Source: http://vajiramias.com/current-affairs/office-of-the-chief-commissioner-for-persons-

with-disabilities-ccpd/5c7a73361d5def14e1472564/

Q20. SWEEKAR (Swachalit Kiraya) and SWAGAT (Swachalit Gate), which was in news recently is

related to:

A. National Common Mobility Card (NCMC) program.

B. Swachh Bharat Abhiyan

C. Bharatmala 2.0 programme

D. Saansad Adarsh Gram Yojana

Answer: A

Page 20: 11th May 2019 Live Test conducted by

Explanation:

Prime Minister Narendra Modi has recently launched One Nation, One Card for transport

mobility. The Indigenous Automatic Fare Collection System based on One Nation One Card

Model i.e. National Common Mobility Card (NCMC) is the first of its kind in India. India’s First

Indigenously Developed Payment Eco-system for transport consisting of NCMC Card, SWEEKAR

(Swachalit Kiraya: Automatic Fare Collection System) and SWAGAT (Swachalit Gate) is based on

NCMC Standards. The customer may use this single card for payments across all segments

including metro, bus, suburban railways, toll, parking, smart city and retail.

The stored value on card supports offline transaction across all travel needs with minimal

financial risk. The service area feature of this card supports operator specific applications such

as monthly passes, season tickets etc. It has been implemented by the Union Ministry of

Housing & Urban Affairs. National Payments Corporation of India (NPCI) has been entrusted to

prepare the standards & specifications of the National Common Mobility Card (NCMC). Centre

for Development of Advance Computing (C-DAC) was entrusted the task of finalization of NCMC

specification for AFC system. Hence, option A. is the correct answer.

Source: http://vajiramias.com/current-affairs/national-common-mobility-

card/5c7e17551d5def14e5662f77/

Q21. Consider the following pairs:

Which of the pairs given above are correctly matched?

A. 1, 2 and 3

B. 1 and 2 only

C. 2 and 3 only

D. None of the above

Page 21: 11th May 2019 Live Test conducted by

Answer: A

Explanation:

Pair (1) is correctly matched: A genetic mutation known as CCR5-delta 32 is responsible for the

two types of HIV resistance that exist. CCR5-delta 32 hampers HIV's ability to infiltrate immune

cells. The mutation causes the CCR5 co-receptor on the outside of cells to develop smaller than

usual and no longer sit outside of the cell. Various mutations of the CCR5 gene are known that

result in damage to the expressed receptor. One of the mutant forms of the gene is CCR5-delta

32, which results from deletion of a particular sequence of 32 base-pairs.

Pair (2) is correctly matched: CRISPR-Cas9 is a genome editing tool that is creating a buzz in the

science world. It is faster, cheaper and more accurate than previous techniques of editing DNA

and has a wide range of potential applications.

Pair (3) is correctly matched: India to Get High-Performing BullSequana Supercomputers worth

Rs 4,500 Crore from France. Atos, a global leader in digital transformation, announced the

installation of its first supercomputer in India. The BullSequana, which is the first

supercomputer to be installed as part of the Indian Government’s National Supercomputing

Mission (NSM), was officially inaugurated at IIT-BHU in Varanasi by the Prime Minister of India

recently.

Source: http://vajiramias.com/current-affairs/ccr5-delta-32/5c7f5e2b1d5def14e147c75e/

Q22. According to an analysis of air quality in several cities around the world, 15 of the top 20

most polluted cities in the world are located in India. Which of the following are the air

pollutants in India?

(1) Sulphur dioxide (SO2)

(2) Nitrogen Oxides (NO2)

(3) Particulate Matter (P.М) 2.5 and 10.

(4) Carbon Monoxide (CO)

(5) Peroxyacetyl nitrate (PAN)

Select the correct answer using the code given below:

A. 1 and 3 only

B. 1, 2, 3 and 4 only

Page 22: 11th May 2019 Live Test conducted by

C. 2, 3, 4 and 5 only

D. 1, 2, 3, 4 and 5

Answer: D

Explanation:

Fifteen of the top 20 most polluted cities in the world are located in India, according to an

analysis of air quality in several cities around the world by IQAir Group, a manufacturer of air-

monitoring sensors as well as purifiers and environmentalist group Greenpeace.

Point (1) is correct: Sulphur dioxide (SO2) is a colorless reactive gaseous air pollutant with a

pungent odor. Sulfur dioxide emissions come from coal-fired power plants as this pollutant is

produced predominantly when coal is burned to generate electricity. India is becoming the

world's top sulphur dioxide emitter.

Point (2) is correct: In atmospheric chemistry, NOₓ is a generic term for the nitrogen oxides that

are most relevant for air pollution, namely nitric oxide and nitrogen dioxide. These gases

contribute to the formation of smog and acid rain, as well as affecting tropospheric ozone.

Vehicles and thermal power plants are the main sources of NOx pollution.

Point (3) is correct: In Delhi, industry and residential activities contribute to 80 per cent of total

PM2.5.Cities in India are also severely exposed to PM10.It is one of the major air pollutants.

Point (4) is correct: Carbon monoxide (CO)—a colorless, odorless, tasteless, and toxic air

pollutant—is produced in the incomplete combustion of carbon-containing fuels, such as

gasoline, natural gas, oil, coal, and wood.

Point (5) is correct: Peroxyacetyl nitrate is a secondary pollutant present in photochemical

smog. peroxyacetyl nitrate, known to irritate the eyes. Smog is a byproduct of different

pollutants like nitrogen oxides, Particulate Matters (PM2.5 and PM10), Volatile Organic

Compounds (VOCs), tropospheric ozone and Peroxyacetyl Nitrate (PAN).All of these substances

are very chemically reactive and are harmful to human and other living things. Hence the

correct option is D..

Source: http://vajiramias.com/current-affairs/air-pollution/5c7f58701d5def14e5666432/

Q23. Consider the following statements with reference to the Generalized System of

Preferences (GSP):

(1) It is a mechanism by which industrially developed countries extend tariff concessions to

certain goods originating in developing countries.

Page 23: 11th May 2019 Live Test conducted by

(2) It is done under the aegis of World Trade Organisation’s (WTO) to promote equitable trade.

Which of the statements given above is/are correct?

A. 1 only

B. 2 only

C. Both 1 and 2

D. Neither 1 nor 2

Answer: A

Explanation:

Statement (1) is correct and (2) is incorrect: The GSP is a non-contractual instrument by which

industrially developed countries extend tariff concession (like reduced MFN tariff or duty free

treatment of eligible products) to certain goods originating in developing countries. The

principles underlying GSP were formally accepted in 1968 during the 2nd UNCTAD Conference

held at New Delhi, and were first put into operation in the year 1971.

The US Generalized System of Preferences (GSP) is a U.S. trade program designed to promote

economic growth in the developing world by providing preferential duty-free entry for up to

4,800 products from 129 designated beneficiary countries and territories. GSP was instituted on

January 1, 1976, by the Trade Act of 1974. India was the largest beneficiary of the programme

in 2017 with USD 5.7 billion in imports to the US given duty-free status and Turkey the fifth

largest with USD 1.7 billion in covered imports. U.S. President Donald Trump notified Congress

his ‘intent to terminate’ preferential trade benefits to India and Turkey under the Generalized

System of Preferences (GSP) eligibility criteria.

Source: http://vajiramias.com/current-affairs/usa-generalized-system-of-preferences-gsp-

programme/5c7f5cbe1d5def14e56664ed/

Q24. Forward Search Experiment (FASER), recently seen in news, is an experiment to:

A. Look for particles associated with mysterious dark matter of universe.

B. Develop potential vaccine against Zika virus.

C. Create Solid Fuel Ducted Ramjet (SFDJ) missile system.

D. Alternative fuels like biofuel to be used as biojet-fuel.

Answer: A

Page 24: 11th May 2019 Live Test conducted by

Explanation:

CERN planning new experiment to look for particles associated with mysterious dark matter of

universe. The CERN, that hosts the world's largest and most powerful particle accelerator,

announced recently that it has approved the experiment designed to look for light and weakly

interacting particles at the Large Hadron Collider (LHC). LHC is a giant lab in a 27-kilometre

tunnel straddling the French-Swiss border. India is an associate member in the LHC project.

The CERN research board has approved the Forward Search Experiment (FASER), giving a green

light to the assembly, installation and use of an instrument that will look for new fundamental

particles at the Large Hadron Collider in Geneva, Switzerland. FASER’s focus is to find light,

extremely weakly interacting particles that have so far eluded scientists, even in the high-

energy experiments conducted at the CERN-operated Large Hadron Collider (LHC), the largest

particle accelerator in the world. Hence, option A. is the correct answer.

Source: http://vajiramias.com/current-affairs/forward-search-experiment-

faser/5c8373d01d5def73164291db/

Q25. Some states in India have recently initiated the cloud seeding projects to tap the clouds. In

this context which of the following chemical(s) is/are used in cloud seeding?

(1) Silver Iodide

(2) Potassium Iodide

(3) Dry Ice

Select the correct answer using the code given below:

A. 1 only

B. 1 and 3 only

C. 2 and 3 only

D. 1, 2 and 3

Answer: D

Explanation:

Cloud seeding is a common term for one form of weather modification. The most common

chemicals used for cloud seeding include silver Iodide, potassium Iodide and dry ice (solid

Page 25: 11th May 2019 Live Test conducted by

carbon dioxide). The process involved in artificial rain-making involves three easy-to-

understand stages.

The first stage is agitation. That is using chemicals to stimulate the air mass upwind of the

target area to rise and form rain clouds. The chemicals used during this stage are calcium

chloride calcium carbide, calcium oxide, a compound of salt and urea, or a compound of urea

and ammonium nitrate. These compounds are capable of absorbing water vapour from the air

mass, thus stimulating the condensation process.

The second stage is called building-up stage. Here the cloud mass is built up using chemicals

such as kitchen salt, the T.1 formula, urea, ammonium nitrate, dry ice, and occasionally also

calcium chloride to increase nuclei which also increase the density of the clouds.

In the third stage of bombardment chemicals such as super-cool agents: silver iodide and dry

ice are used to reach the most unbalanced status which builds up large beads of water (Nuclei)

and makes them fall down as raindrops.

Karnataka state government is planning to carry out cloud seeding operations to enhance

rainfall during the monsoons of 2019 and 2020 as 176 taluks in the State are reeling under

drought. The department is aiming to replicate the Rs. 35-crore ‘Varshadhare’ project two years

ago that was called a success by an independent evaluation committee.

Source: http://vajiramias.com/current-affairs/cloud-seeding/5c862d5c1d5def18d99ed610/

Q26. Which of the following item(s) has/have recently got the Geographical Identification Tag in

India?

(1) ‘Sirsi Supari’ of Uttara Kannada

(2) Idukki’s Marayoor jaggery

Page 26: 11th May 2019 Live Test conducted by

(3) Erode turmeric

Select the correct answer using the code given below:

A. 1 only

B. 1 and 3 only

C. 2 and 3 only

D. 1, 2 and 3

Answer: D

Explanation:

Point (1) is correct: It is for the first time in the arecanut sector ‘Sirsi Supari’ grown in Uttara

Kannada has received the Geographic Indication (GI) tag. Sirsi Supari is cultivated in Yellapura,

Siddapura and Sirsi taluks in Uttara Kannada. Totgars’ Cooperative Sale Society Ltd., Sirsi, is the

registered proprietor of the GI. The Registrar of Geographical Indications, under the Union

government, Chennai issued the certificate to the society on March 4, 2019. ‘Sirsi Supari’ is

used both as ‘chali’ (white arecanut) and red arecanut.

Point (2) is correct: Marayur organic jaggery produced at Marayur in the hilly district of Idukki in

Kerala has bagged Geographical Indication tag. Marayur jaggery is known especially for its

organic and traditional production using the sugar cane cultivated in the region. It does not

contain any chemicals and is high in iron and calcium. The Centre granted the GI tag

considering various aspects of its manufacturing and the peculiarities of the region.

Point (3) is correct: Recently, Erode manjal gets GI tag. With the GI tag, turmeric cultivated in

Kodumudi, Sivagiri, Bhavani, Gobichettipalayam, Anthiyur, Chennampatti , Sathyamangalam

and Thalavady of Erode district, parts of Coimbatore and whole of Tirupur will be recognised for

its unique qualities derived from its place of origin.

Page 27: 11th May 2019 Live Test conducted by

Source: http://vajiramias.com/current-affairs/sirsi-supari/5c8788131d5def18da3daec9/

Q27. Which of the following pairs is/are correctly matched?

Select the correct answer using the code given below:

A. 1 only

Page 28: 11th May 2019 Live Test conducted by

B. 1 and 3 only

C. 2 and 3 only

D. 1, 2 and 3

Answer: A

Explanation:

Pair (1) is correctly matched: Scientists, using NASA’s Lunar Reconnaissance Orbiter (LRO), have

observed water molecules moving around the dayside of the Moon. Measurements from the

Lyman Alpha Mapping Project (LAMP) instrument aboard the LRO of the sparse layer of

molecules temporarily stuck to the surface helped characterise lunar hydration changes over

the course of a day. The latest research revealed the amount of energy needed to remove

water molecules from lunar materials, helping scientists understand how water is bound to

surface materials. The Lunar Reconnaissance Orbiter (LRO) is a NASA robotic spacecraft

currently orbiting the Moon in an eccentric polar mapping orbit.

Pair (2) is not correctly matched: The Lunar Crater Observation and Sensing Satellite (LCROSS)

was a robotic spacecraft operated by NASA. The mission was conceived as a low-cost means of

determining the nature of hydrogen detected at the polar regions of the Moon.

Pair (3) is not correctly matched: KAGUYA is a Japanese Space Agency (JAXA) lunar orbiter

mission. KAGUYA, initially named SELENE (SELenological and ENgineering Explorer) was a

Japanese mission to map the Moon's surface and study lunar origins and evolution. It also

carried two smaller satellites to study the Moon's gravity and ionosphere. This mission served

as an engineering test for future deep space missions.

Source: http://vajiramias.com/current-affairs/lunar-reconnaissance-orbiter-

lro/5c878bed1d5def18da3daf98/

Q28. Which of the following pairs is/are correctly matched?

Page 29: 11th May 2019 Live Test conducted by

Select the correct answer using the code given below:

A. 2 only

B. 1 and 3 only

C. 2 and 3 only

D. 1, 2 and 3

Answer: A

Explanation:

Pair (1) is not correctly matched: Christchurch, known for its English heritage, is located on the

east coast of New Zealand’s South Island. Recently Christchurch is in news due to two

consecutive terrorist mass shootings at Al Noor Mosque and the Linwood Islamic Centre.

Pair (2) is correctly matched: Xinjiang, officially the Xinjiang Uygur Autonomous Region (XUAR),

is a provincial-level autonomous region of China in the northwest of the country. It is the largest

of China's administrative regions.

Pair (3) is not correctly matched: Jalalabad is a city in eastern Afghanistan. It is the capital of

Nangarhar Province. Jalalabad is located at the junction of the Kabul River and the Kunar River.

Page 30: 11th May 2019 Live Test conducted by

Source: http://vajiramias.com/current-affairs/us-taliban-doha-

talks/5c88dc281d5def4480e4c7dc/

http://vajiramias.com/current-affairs/xinjiang/5c90a3431d5def05bf423493/

Q29. Recently, a species of wood snake “Xylophis indicus” that wasn’t seen for 140 years has

resurfaced in a survey conducted by scientists. It is endemic to:

A. Meghamalai forests and Periyar Tiger Reserve area.

B. Jim Corbett National Park

C. Seshachalam Hills and Biosphere reserve area.

D. Khangchendzonga Hills and Biosphere reserve area.

Answer: A

Explanation:

A species of wood snake – Xylophis indicus – that wasn’t seen for 140 years has resurfaced in a

survey conducted by scientists in the Meghamalai Wildlife Sanctuary.The species is endemic to

the Meghamalai forests and Periyar Tiger Reserve area. The local population of wood snakes

Page 31: 11th May 2019 Live Test conducted by

was last spotted and recorded by British military officer and naturalist Colonel Richard Henry

Beddome in 1878, who went on to describe it as a new species, Xylophis indicus. The specimens

he collected was labelled as being from “the dense heavy evergreen forests on the mountains

at the south of the Cumbum valley, Madurai”. This locality (alternative spelling: Kambam) is

now probably within the Teni District of Tamil Nadu, close to the state border with Kerala. The

findings of the surveys, conducted over two years (2014-2016), were published in the Journal of

the Bombay Natural History Society last month. The snake he discovered was 235 mm long and

uniformly dark brown. Hence, option A. is the correct answer.

Source: http://vajiramias.com/current-affairs/xylophis-indicus/5c88d8e11d5def447ea80102/

Q30. With reference to “Sunspots”, consider the following statements:

(1) They are regions where the solar magnetic field is very weak.

(2) In visible light, they appear darker than their surroundings because they are a few thousand

degrees cooler than their surroundings.

Which of the statements given above is/are correct?

A. 1 only

B. 2 only

C. Both 1 and 2

D. Neither 1 nor 2

Answer: B

Explanation:

Statement (1) is incorrect: Sunspots are regions where the solar magnetic field is very strong.

Statement (2) is correct: In visible light, sunspots appear darker than their surroundings

because they are a few thousand degrees cooler than their surroundings.

Sunspots do not appear everywhere on the Sun. They are usually concentrated in two bands,

about 15 - 20 degrees wide in latitude, that go around the Sun on either side of the solar

equator. The amount of magnetic flux that rises up to the Sun’s surface varies with time in a

cycle called the solar cycle. This cycle lasts 11 years on average. This cycle is sometimes referred

to as the sunspot cycle. Solar Tsunami is a hot plasma wave racing across the sun’s surface.

Solar Tsunami are nothing but a Moreton Wave. Recently, a group of solar physicists from India

have suggested that a “solar tsunami” triggers the new sunspot cycle, after the old one ends.

Page 32: 11th May 2019 Live Test conducted by

Source: http://vajiramias.com/current-affairs/sunspot-cycle/5c8e31011d5def6121836a95/

Q31. Which of the following Scorpene class stealth submarines are built under the Project 75?

(1) INS Karanj

(2) INS Vikrant

(3) INS Khanderi

(4) INS Vela

Select the correct answer using the code given below:

A. 1 and 2 only

B. 1 and 3 only

C. 2, 3 and 4 only

D. 1, 3 and 4 only

Answer: D

Explanation:

Six Scorpene class submarines are being built under Project 75. The submarines are designed by

French naval defence and energy group DCNS and manufactured by the Mazagon Dock

Shipbuilders Limited (MDSL), Mumbai under a 2005 contract worth $3.75 billion.

(i) INS Kalvari: It was commissioned in December 2017.

(ii) INS Khanderi: It was launched in January 2017, is currently undergoing series of trials and is

expected to be commissioned by May 2019.

(iii) INS Karanj: It was launched on 31 st January 2018.

(iv) INS Vela: It was launched in May 2019.

(v) INS Vagir and Vagsheer: The remaining two submarines are in the series are in advanced

stages of manufacturing and trials.

The entire project is expected to be completed by 2020. The fifth submarine Vagir, is in the final

stages of being booted together. The ‘Boot Together’ is where the five separate sections are

welded together to form the submarine.

Page 33: 11th May 2019 Live Test conducted by

INS Vikrant is the first aircraft carrier built in India and the first aircraft carrier built by Cochin

Shipyard in Kochi for the Indian Navy. Hence option D. is correct.

Source: http://vajiramias.com/current-affairs/scorpene-

submarines/5c8f6f591d5def05bf41e51a/

Q32. The selection of the Chairperson and the members of Lokpal shall be through a Selection

Committee would comprise of:

(1) Prime Minister

(2) Chief Justice of India

(3) Lok Sabha Speaker

(4) Leader of the Opposition

Select the correct answer using the code given below:

A. 1 and 2 only

B. 1, 2 and 3 only

C. 2, 3 and 4 only

D. 1, 2, 3 and 4

Answer: D

Explanation:

The Lokpal and Lokayuktas Act 2013 allows for setting up of anti-corruption ombudsman called

Lokpal at the Centre. A Lokayukta is to be appointed in every state within one year of the

passing of the Act (but several states are yet to appoint such an institution).

Page 34: 11th May 2019 Live Test conducted by

Selection procedure of Lokpal: The five-member selection committee comprises the following –

Prime Minister (chairperson), Lok Sabha Speaker, Leader of the Opposition, Chief Justice of

India and an eminent jurist nominated by the President. The Lokpal will consist of a chairperson

and a maximum of eight members. Chairperson should have been a Chief Justice of India, or is

or has been a judge of the Supreme Court, or an eminent person who fulfils eligibility criteria as

specified. 50% of the members are to be judicial members, provided that not less than 50% of

the members belong to the Scheduled Castes, Scheduled Tribes, OBCs, minorities, and women.

Hence, option D. is the correct answer.

Source: http://vajiramias.com/current-affairs/pinaki-chandra-

ghose/5c8f73131d5def05bc977473/

Q33. With reference to the notices issued by Interpol, which of the following pairs is/are

correctly matched?

Select the correct answer using the code given below:

A. 1 only

B. 1 and 2 only

C. 2 and 3 only

Page 35: 11th May 2019 Live Test conducted by

D. 1, 2 and 3

Answer: A

Explanation:

Interpol is ‘NOT’ a unit or part of united nation system. It is an independent international

organization. It issues 8 types of notices (7 of which are colour-coded) which are in the form of

alert/requests allowing police in member countries to share critical crime-related information.

1. Red Notice: To seek the location and arrest of a person wanted by a judicial jurisdiction or an

international tribunal with a view to his/her extradition. It is the ‘closest instrument to an

international arrest warrant’.

2. Blue Notice: To locate, identify or obtain information on a person of interest in a criminal

investigation.

3. Green Notice: To warn about a person’s criminal activities if that person is considered to be a

possible threat to public safety.

4. Yellow Notice: To locate a missing person or to identify a person unable to identify

himself/herself.

5. Black Notice: To seek information on unidentified bodies.

6. Orange Notice: To warn of an event, a person, an object or a process representing an

imminent threat and danger to persons or property.

7. Purple Notice: To provide information on modus operandi, procedures, objects, devices or

hiding places used by criminals.

8. Interpol-UNSC Special Notice: To inform Interpol’s members that an individual or an entity is

subject to UN sanctions.

Page 36: 11th May 2019 Live Test conducted by

Source: http://vajiramias.com/current-affairs/interpol-red-

notice/5c9613161d5def2fdf4d79e2/

Q34. Consider the following countries:

(1) Canada

(2) Italy

(3) United States

(4) India

(5) Russia

Which of the above countries are among the members of Group of Seven (G-7)?

A. 1, 2 and 3 only

B. 2, 3 and 4 only

C. 1, 2, 3 and 5 only

D. 2, 3, 4 and 5 only

Answer: A

Explanation:

The Group of Seven (G-7) is a forum of the world’s seven most industrialized economies. It

consists of Canada, France, Germany, Italy, Japan, the United Kingdom, and the United States.

The G-7 meets on an annual basis in a rotating member country to discuss international

economic and monetary issues. The European Union is also represented at the G7 summit. The

Page 37: 11th May 2019 Live Test conducted by

G-6 was formed in 1975 and consisted of France, Germany, Italy, Japan, the United States and

the United Kingdom; Canada was invited to join the group in 1976 creating the G-7.

In 1998, Russia was added to the G-7 as a full member, making the group of world leaders the

G-8. The acceptance of Russia created the G-8 from 1998 to 2014. In 2014, Russia was

suspended from the group after the annexation of Crimea and tensions in Ukraine. Hence,

option A. is the correct answer.

Italy is projected to be the first G7 nation to officially endorse China’s Belt and Road Initiative

(BRI).

Source: http://vajiramias.com/current-affairs/belt-and-road-initiative-bri-

italy/5c9616f51d5def2fdbeade77/

Q35. ‘Refuse to Ride’ campaign, recently seen in news, is launched by which of the following?

A. The World Wide Fund

B. The Ministry of Environment, Forest and Climate Change

C. Wildlife SOS

D. TRAFFIC, the Wildlife Trade Monitoring Network

Answer: C

Explanation:

Gatimaan Express is now actively promoting Refuse To ride campaign initiated by Wildlife SOS

as a part of responsible tourism. In 2018, the Animal Welfare Board of India, Ministry of the

Environment and Forests, published a report about the dire conditions of the elephants used

for tourist rides at Amer Fort. In this background Wildlife SOS launched its ‘Refuse to Ride’

campaign to open the eyes of the public to the harsh realities surrounding traditional methods

that involve training elephants for tourist rides by using painful and cruel methods.

Wildlife SOS (WSOS) is a conservation non-profit organization in India. Its Primary objective is to

rescue and rehabilitate wildlife in distress in the country. It is internationally known for ground

breaking work of eradicating the cruel and barbaric practice of Dancing Bears from the streets

of India. Hence, option C. is the correct answer.

Source: http://vajiramias.com/current-affairs/refuse-to-ride/5c9761131d5def0a1ca88d64/

Q36. Which of the following pairs are correctly matched?

Page 38: 11th May 2019 Live Test conducted by

Select the correct answer using the code given below:

A. 1 and 2 only

B. 1 and 3 only

C. 2 and 3 only

D. 1, 2 and 3

Answer: A

Explanation:

Pair (1) is correctly matched: The famous biennial Thakurani Jatra festival is in Odisha’s

Berhampur. The celebrations will start from March 29 and continue for 32 days. The Month-

long festival is held once in two years. The Mother Goddess is worshipped in the Maa Budhi

Thakurani Temple in Berhampur town in Odisha. It is the most important ritual of the festival,

which begins on the third day. In this the garland dedicated to Goddess Budhi Thakurani is put

inside a decorated earthen pot, called Ghata.

Pair (2) is correctly matched: Sammakka Saralamma Jatara or Medaram Jatara is a festival of

honouring the Hindu goddesses, celebrated in the state of Telangana, India. It is the biggest

Tribal festival in Asia which is attended by one crore people on an average. Central government

is likely to declare Medaram’s Sammakka-Sarakka/Saralamma Jatara a national festival this

year. It is held by forest dwelling Koya tribe of Telangana and surrounding States. It is a massive

event which is held bi-annually in Jayashankar Bhupalpally district to honour the twin

goddesses Sammakka and her daughter Sarakka.

Page 39: 11th May 2019 Live Test conducted by

Pair (3) is not correctly matched: The Nagoba jatara — the annual religious event of the

Buigoita branch of Mesram clan of the Raj Gond adivasis in Adilabad district, Telangana. It is the

second biggest tribal carnival.

Source: http://vajiramias.com/current-affairs/thakurani-jatra-

festival/5c88dda51d5def447ea801e2/

Q37. Which of the following Conventions on Diplomatic Relations is an international treaty that

defines a framework for diplomatic relations between independent countries?

A. Stockholm Convention

B. Bonn Convention

C. Basel Convention

D. Vienna Convention

Answer: D

Explanation:

Diplomatic protection is a rarely-used mechanism allowing nations to seek protection on behalf

of its citizens on the grounds that they have been wronged by another state. It can be traced

back to Article 29 of Vienna Convention on Diplomatic Relations, 1961 which says that

“Diplomats must not be liable to any form of arrest or detention. The Vienna Convention on

Diplomatic Relations of 1961 is an international treaty that defines a framework for diplomatic

relations between independent countries. Diplomats are immune from civil or criminal

prosecution in the host country”. Hence, option D. is the correct answer.

Source: http://vajiramias.com/current-affairs/diplomatic-

immunity/5c862a841d5def18dd5326df/

Q38. According to an UN Office for Disaster Risk Reduction (UNISDR) report, India has been

ranked as the world's most disaster-prone country for displacement of residents. Which of the

following are the examples of atmospheric disasters?

(1) Lightning

(2) Tropical cyclone

(3) Earthquake

(4) Tsunami

Page 40: 11th May 2019 Live Test conducted by

Select the correct answer using the code given below:

A. 1 and 2 only

B. 2 and 4 only

C. 1, 2 and 3 only

D. 1, 2, 3 and 4

Answer: A

Explanation:

Natural Disasters are broadly categorized as −Atmospheric Disasters, Terrestrial Disasters,

Aquatic Disasters and Biological Disasters. Atmospheric disasters include blizzard,

thunderstorm, lightning, tropical cyclone, tornado, drought, hailstorm, frost, heat wave, cold

waves, etc.

Terrestrial disasters include earthquake, volcanic eruption, landslide, avalanches, subsidence,

etc.

Aquatic disasters include flood, tidal waves, storm surge, tsunami, etc.

Biological disasters include fungal, bacterial, and viral diseases (e.g. bird flu, dengue, etc.).

The Odisha government has decided to have lightning protection system in multipurpose

shelters across 25 districts as lightning has emerged as the leading cause behind natural deaths

across the State. In the first phase, 640 shelters would be taken up with funding support from

the World Bank under the National Cyclone Risk Mitigation Project.Hence, option A. is the

correct answer.

Source: http://vajiramias.com/current-affairs/lightning/5c7ce2841d5def14e5660123/

Q39. With respect to the recent expansion in India's "Standards & Labelling (Star Rating)

program", consider the following statements:

(1) It is mandatory for domestic consumers to shift towards energy efficient microwave ovens

and washing machines.

(2) It is implemented by the energy efficiency services limited (EESL).

Which of the statements given above is/are correct?

A. 1 only

Page 41: 11th May 2019 Live Test conducted by

B. 2 only

C. Both 1 and 2

D. Neither 1 nor 2

Answer: D

Explanation:

Statement (1) is incorrect: Union Ministry of Power launched expanded its Standards &

Labelling (Star Rating) program for Energy Efficient for Appliances to cover the Microwave

Ovens and Washing Machines (with revised parameters) in the country. It aims to create

awareness amongst the domestic consumers to encourage transition towards energy efficient

microwave ovens and Washing machines. Initially, the program for Microwave Ovens and

Washing Machines will be implemented on a ‘VOLUNTARY’ basis and will be valid up to 31st

December 2020.

Statement (2) is incorrect: It is implemented by Bureau of Energy Efficiency (BEE).

Source: http://vajiramias.com/current-affairs/star-rating-programme-for-microwave-ovens-

and-washing-machines/5c8379931d5def7316429415/

Q40. With reference to “GRAPES-3 (Gamma Ray Astronomy PeV EnergieS phase-3)”, consider

the following statements:

(1) It is located at Ooty, India to probe acceleration of cosmic rays.

(2) It is an initiative taken by the Indian Institute of Technology, Madras.

Which of the statements given above is/are correct?

A. 1 only

B. 2 only

C. Both 1 and 2

D. Neither 1 nor 2

Answer: A

Explanation:

Page 42: 11th May 2019 Live Test conducted by

For the first time in the world, researchers at the GRAPES-3 muon telescope facility in Ooty

have measured the electrical potential, size and height of a thundercloud that passed overhead

on December 1, 2014.

Statement (1) is correct- GRAPES-3 (Gamma Ray Astronomy PeV EnergieS phase-3) experiment

is located at Ooty, India. It aims to probe acceleration of cosmic rays in the following four

astrophysical settings:

~100 MeV in atmospheric electric fields through muons,

~10 GeV in Solar system through muons,

~1 PeV in our galaxy through nuclear composition of cosmic rays,

~100 EeV in nearby universe through measurement of diffuse γ-ray flux.

Statement (2) is incorrect- It started as a collaboration of the Tata Institute of Fundamental

Research, Mumbai, India and the Osaka City University, Osaka, Japan. At present many

institutions from India and Japan are in collaboration.

Source: http://vajiramias.com/current-affairs/grapes-3/5c9761c11d5def0a17f2f33e/

Q41. Which of the following countries borders the ‘Golan Heights’, which was in news recently?

(1) Israel

(2) Lebanon

(3) Jordan

(4) Egypt

Select the correct answer using the code given below:

A. 1 and 2 only

B. 2, 3 and 4 only

C. 1, 2 and 3 only

D. 1, 2, 3 and 4

Answer: C

Explanation:

Page 43: 11th May 2019 Live Test conducted by

The Golan Heights is a rocky plateau in south-western Syria, borders Israel, Lebanon and

Jordan. The area is hilly and elevated, overlooking the Jordan Rift Valley which contains the Sea

of Galilee and the Jordan River, and is itself dominated by the Mount Hermon. More than

40,000 people live on the Israeli-occupied Golan, more than half of them Druze residents. The

Druze are an Arab minority who practice an offshoot of Islam. After annexing the Golan, Israel

gave the Druze the option of citizenship, but most rejected it and still identify as Syrian. The

Golan Heights were part of Syria until 1967, when Israel captured most of the area in the Six

Day War, occupying it and annexing it in 1981.Syria tried to regain the Heights in the 1973

Middle East war, but was thwarted. Israel and Syria signed an armistice in 1974 and the Golan

had been relatively quiet since. That unilateral annexation was not recognized internationally,

and Syria demands the return of the territory.

Recently US President Donald Trump has said that the United States should back Israeli

sovereignty over the Golan Heights, seized from Syria in 1967.

Source: http://vajiramias.com/current-affairs/golan-heights/5c9765941d5def0a1ca88e7c/

Q42. Which of the following documents from India are included in the UNESCO's Memory of

the World Programme?

(1) Maitreyayvarakarana

(2) Rigveda Saiva Manuscript in Pondicherry

(3) The I.A.S. Tamil Medical Manuscript Collection

(4) Astasahasrika Prajnaparamita

Page 44: 11th May 2019 Live Test conducted by

Select the correct answer using the code given below:

A. 1 and 2 only

B. 1 and 3 only

C. 1, 2 and 3 only

D. 1, 2, 3 and 4

Answer: C

Explanation:

UNESCO's Memory of the World Programme is an international initiative launched to safeguard

the documentary heritage of humanity against collective amnesia, neglect, the ravages of time

and climatic conditions, and willful and deliberate destruction. It calls for the preservation of

valuable archival holdings, library collections and private individual compendia all over the

world for posterity. It is different from UNESCO World Heritage and UNESCO Intangible

Heritage.

India has following documentary heritage in the list –

(i) Archives of the Dutch East India Company

(ii) Gilgit Manuscrpit

(iii) Laghu-kalachakra-tantra-rajatika

(iv) Maitreyayvarakarana

(v) Rigveda Saiva Manuscript in Pondicherry

(vi) Shantinatha Charitra

(vii) Tarikh-E-Khandan-E-Timuriyah

(viii) The I.A.S. Tamil Medical Manuscript Collection

Point (1) is correct: Maitreyayvarakarana is a very short text, written on palm leaf in the mixed

character of Kutila and Ranjana script, a manuscript of the Pala period (at the time of

Gopaladeva, 57th regna; year, approx. 10th century A.D.) is in the possession of the Asiatic

Society, Kolkata. It is believed, Maitreya, the future prophet will eventually appear on earth as

enlightened Buddha and teach the pure dharma. The text was written in a period when

Buddhist literature faced a transition from the Sthaviravada to Mahayana school. Documentary

Page 45: 11th May 2019 Live Test conducted by

heritage submitted by India and recommended for inclusion in the Memory of the World

Register in 2017.

Point (2) is correct: Rigveda Saiva Manuscript in Pondicherry: Within a collection of 11 000

manuscripts that concern mainly the religion and worship of the Hindu God Siva, is included the

largest collection in the world of manuscripts of texts of the Śaiva Siddhānta. In the 10th

century CE, this religious tradition, a major current of Hinduism, was spread right across the

Indian subcontinent and beyond, as far as Cambodia in the East. It long represented the

mainstream of Tantric doctrine and worship and appears to have influenced every Indian

theistic tradition. Its surviving texts, the majority of them unpublished, range from the 6th

century CE to the colonial period. Documentary heritage submitted by India and recommended

for inclusion in the Memory of the World Register in 2005.

Point (3) is correct: The I.A.S. Tamil Medical Manuscript Collection: Mostly Tamil Medical

Manuscripts preserved at the Institute of Asian Studies reflect the ancient system of medicine,

practised by yogis. This system explains the methods of obtaining medicines from herbs, herbal

roots, leaves, flowers, barks, fruits etc. The proportions of the ingredients as well as the specific

processes are explained in detail. Documentary heritage submitted by India and recommended

for inclusion in the Memory of the World Register in 1997.

Point (4) is incorrect: Prajnaparamita: The body of sutras and their commentaries that

represents the oldest of the major forms of Mahayana Buddhism, one that radically extended

the basic concept of ontological voidness. The name denotes the female personification of the

literature or of wisdom, sometimes called the Mother of All Buddhas. In

the Prajnaparamita texts, prajna (wisdom), an aspect of the original Eightfold Path, has become

the supreme paramita (perfection) and the primary avenue to nirvana. The main creative

period of Prajnaparamita thought extended from perhaps 100 BCE to 150 CE. The best-known

work from this period is the Astasahasrika Prajnaparamita (Eight Thousand-Verse

Prajnaparamita). However it is not included in the UNESCO's Memory of the World Programme.

Source: http://vajiramias.com/current-affairs/tripitaka/5c9764151d5def0a17f2f371/

Q43. What of the following best describes the term “Ectogenesis”, which was recently seen in

news?

A. It is the growth of an organism in an artificial environment outside the body in which it would

normally be found.

B. It is a family of encrypting ransomware that was first discovered in 2016.

Page 46: 11th May 2019 Live Test conducted by

C. It is an enzyme-based chemotherapeutic agent used to treat acute lymphoblastic leukemia

(ALL).

D. None of the above

Answer: A

Explanation:

Ectogenesis is the growth of an organism in an artificial environment outside the body in which

it would normally be found, such as the growth of an embryo or fetus outside the mother’s

body, or the growth of bacteria outside the body of a host. The term was coined by British

scientist J.B.S. Haldane in 1924.

For the first time ever in the world, scientists have come up with an “artificial womb” which can

support extremely premature lamb foetuses, and believe that this can one day save the lives of

human babies. Artificial wombs are mechanisms that are used to grow an embryo outside of

the body of a female. This technology could bring hope to parents who before could not have

children. It could also open the door for mothers to choose to have their babies outside their

bodies, to avoid the stress of childbirth. Hence, option A. is the correct answer.

Source: http://vajiramias.com/current-affairs/artificial-womb/5c9c99891d5def24148b9df5/

Q44. Which of the following is/are the allotrope(s) of carbon?

(1) Graphene

(2) Diamond

(3) Lonsdaleite

Select the correct answer using the code given below:

A. 1 only

B. 2 only

C. 2 and 3 only

D. 1, 2 and 3

Answer: D

Explanation:

Page 47: 11th May 2019 Live Test conducted by

Researchers at National Physical Laboratory (CSIR-NPL) have designed a low-pressure chemical

vapour deposition (LPCVD) device that allows high quality, single-layer graphene measuring 4

inches in length and 2 inches in width to be grown. The LPCVD device developed indigenously

by CSIR-NPL costs about Rs.5,00,000, which is one-tenth of the imported ones.

Point (1) is correct: Graphene is a form of carbon and a super-strong, ultra-light material

discovered in 2004. Graphene consists of a single layer of carbon atoms arranged in a

hexagonal lattice, each atom bound to its neighbours by chemical bonds. The elasticity of these

bonds produces resonant vibrations known as phonons. It enables flexible electronic

components, enhances solar cell capacity, and promises to revolutionise batteries.

Point (2) is correct: Carbon has several allotropes, or different forms in which it exists. Diamond

is the hardest naturally occurring substance, and graphite is one of the softest known

substances. Diamond is transparent, the ultimate abrasive, and can be an electrical insulator

and thermal conductor. Conversely, graphite is opaque, a very good lubricant, a good

conductor of electricity, and a thermal insulator.

Point (3) is correct: Lonsdaleite, also called hexagonal diamond in reference to the crystal

structure, is an allotrope of carbon with a hexagonal lattice. In nature, it forms when meteorites

containing graphite strike the Earth. Hence, option D. is the correct answer.

Source: http://vajiramias.com/current-affairs/low-pressure-chemical-vapour-deposition-

lpcvd-device/5c84c8051d5def029667633f/

Q45. Consider the following statements:

(1) The U.S.–Russian boundary extends through this strait.

(2) It is the only marine gateway between the icy Arctic and the Pacific Ocean.

(3) It separates the continents of Asia and North America at their closest point.

(4) The International Date Line passes through this strait.

Above statements describes which of the following straits?

A. Strait of Malacca

B. Strait of Hormuz

C. Formosa Strait

D. Bering Strait

Page 48: 11th May 2019 Live Test conducted by

Answer: D

Explanation:

Bering Strait linking the Arctic Ocean with the Bering Sea and separating the continents

of Asia and North America at their closest point. There are numerous islands in the strait,

including the two Diomede Islands and St. Lawrence Island. The U.S.–Russian boundary extends

through the strait. The International Date Line passes through this strait. It is the only marine

gateway between the icy Arctic and the Pacific Ocean. At its narrowest point, the strait is only

55 miles wide. Each spring, one of the largest wildlife migrations on Earth passes through this

narrow gateway to reach the Arctic’s incredibly nutrient-rich and productive waters. Hence,

option D. is the correct answer.

Source: http://vajiramias.com/current-affairs/mueller-report/5ca099aa1d5def0603ed8929/

Q46. Who among the following national leaders had been associated with the Indian Home

Rule Society (IHRS), an Indian organisation founded in London in 1905 to promote the cause of

self-rule in British India?

(1) Shyamji Krishna Varma

(2) Bhikaji Cama

(3) Dadabhai Naoroji

(4) Bal Gangadhar Tilak

Page 49: 11th May 2019 Live Test conducted by

Select the correct answer using the code given below:

A. 1, 2 and 3 only

B. 1 and 2 only

C. 1, 2 and 4 only

D. 1, 2, 3 and 4

Answer: A

Explanation:

Prime Minister recently spoke about the contributions of remembered Shyamji Krishna

Varma(1857 – 1930) on his death anniversary. He was an Indian freedom fighter, lawyer and

journalist.

Indian Home Rule Society (IHRS) was an Indian organisation founded in London in 1905 that

sought to promote the cause of self-rule in British India. The organisation was founded by

Shyamji Krishna Varma, with support from Bhikaji Cama, Dadabhai Naoroji and S.R. Rana.

Hence option A. is correct.

Shyamji Krishna Varma founded India House in London. It was a student residence that existed

between 1905 and 1910 which was opened to promote nationalist views among Indian

students in Britain. He founded Indian Sociologist in London. The monthly Indian Sociologist

became an outlet for nationalist ideas.

Source: http://vajiramias.com/current-affairs/shyamji-krishna-

varma/5ca07d761d5def05fd1b7dfe/

Q47. Which of the following is a famous temple in India to be named after a craftsman who

built it, rather than the king who commissioned it or its presiding deity?

A. Chennakeshava Temple, Karnataka

B. Ramappa temple, Telangana

C. Tugnath Temple, Uttarakhand

D. Konark Sun Temple, Odisha

Answer: B

Explanation:

Page 50: 11th May 2019 Live Test conducted by

Ramappa temple is in the waiting list to be recognized as a standalone UNESCO site, the first in

Telangana.

Ramappa Temple is also known as the Ramalingeswara temple. The temple is named after the

sculptor Ramappa, who built it, and is perhaps the only temple in India to be named after a

craftsman who built it, rather than the king who commissioned it or its presiding deity. Hence

option B. is correct.

It lies in a valley at Palampet village, near Warangal, Telangana. An inscription in the temple

dates it to the year 1213 AD, during the period of the Kakatiya Dynasty rule. The temple is a

Shivalaya, where Lord Ramalingeswara is worshipped. It stands on a 6 ft high star-shaped

platform. The main structure is in reddish sandstone. The temple columns are made of black

basalt and are carved as mythical animals or female dancers or musicians, and are the

masterpieces of Kakatiya art, notable for their delicate carving, sensuous postures and

elongated bodies and heads.

Source: http://vajiramias.com/current-affairs/ramappa-temple/5ca086581d5def05ffdf2f24/

Q48. Which of the following is/are the application(s) of Benzoic acid?

(1) Food preservation

(2) Medicine for fungal/bacterial infection

(3) Manufacture of various cosmetics, dyes and plastics

Select the correct answer using the code given below:

A. 1 only

B. 1 and 2 only

Page 51: 11th May 2019 Live Test conducted by

C. 2 and 3 only

D. 1, 2 and 3

Answer: D

Explanation:

Benzoic acid (or C 6 H 5 COOH), is a colourless crystalline solid and a simple aromatic carboxylic

acid. Benzoic acid is used as a food preservative and medicine for fungal/bacterial infection. It is

also used to manufacture various cosmetics, dyes, plastics, and insect repellents. Among the

derivatives of benzoic acid are sodium benzoate, a salt used as a food preservative; benzyl

benzoate, an ester used as a miticide; and benzoyl peroxide, used in bleaching flour and in

initiating chemical reactions for preparing certain plastics. Hence, option D. is the correct

answer.

Source: http://vajiramias.com/current-affairs/benzoic-acid/5ca0960e1d5def0603ed87ea/

Q49. With respect to “Mission Shakti” Anti-Satellite (A-SAT) missile test, consider the following

statements:

(1) Under this mission a Ballistic Missile Defence (BMD) Interceptor Missile engaged an Indian

orbiting target satellite in High Earth Orbit (HEO) in a ‘Hit to Kill’ mode.

(2) India is only the 5th country after the US, Israel, Russia and China to acquire such a

specialised and modern capability.

Which of the statements given above is/are correct?

A. 1 only

B. 2 only

C. Both 1 and 2

D. Neither 1 nor 2

Answer: D

Explanation:

Defence Research and Development Organisation (DRDO) successfully conducted an Anti-

Satellite (A-SAT) missile test ‘Mission Shakti’ from the Dr AP J Abdul Kalam Island in Odisha.

Page 52: 11th May 2019 Live Test conducted by

Statement (1) is incorrect: A DRDO-developed Ballistic Missile Defence (BMD) Interceptor

Missile successfully engaged an Indian orbiting target satellite in Low Earth Orbit (LEO) in a ‘Hit

to Kill’ mode. The interceptor missile was a three-stage missile with two solid rocket boosters.

Statement (2) is incorrect: The test has demonstrated the Nation’s capability to defend its

assets in outer space. India is only the 4th country after the US, Russia and China to acquire

such a specialised & modern capability. Entire effort is indigenous.

A low Earth orbit is normally at an altitude of less than 1000 km and could be as low as 160 km

above the Earth. These orbits are used for remote sensing, military purposes and for human

spaceflight. The International Space Station is in low Earth orbit.

Source: http://vajiramias.com/current-affairs/mission-shakti/5c9c6f531d5def2413acb2b0/

Q50. Which of the following is not correct about “YUva VIgyani KAryakram (YUVIKA)”

programme?

A. Its objective is to impart basic knowledge on Space Technology, Space Science and Space

Applications to the school children.

B. Students belong to the rural area have been given special weightage in the selection criteria.

C. The programme has been launched by the Ministry of Human Resource Development.

D. Those who have just finished 9th standard and waiting to join 10th standard will be eligible

for the programme.

Answer: C

Page 53: 11th May 2019 Live Test conducted by

Explanation:

Statement C. is incorrect: YUva VIgyani KAryakram (YUVIKA), also known as “Young Scientist

Programme” is launched by Indian Space Research Organisation (ISRO) in March 2019.

Its objective is to impart basic knowledge on Space Technology, Space Science and Space

Applications to the school children with the intent of arousing their interest in the emerging

areas of Space activities. The residential training programme will be of around two weeks

duration during summer holidays. It is proposed to select 3 students each from each State/

Union Territory to participate in this programme every year covering CBSE, ICSE and State

syllabus. Those who have just finished 9th standard (in the academic year 2018-19) and waiting

to join 10th standard (or those who have started 10th Std just now) will be eligible for the

programme. The selection is based on the academic performance and extracurricular activities.

Students belong to the rural area have been given special weightage in the selection criteria.

Source: http://vajiramias.com/current-affairs/yuvika/5c9a09c31d5def136b6c3bc8/

Q51. Which of the following missions have been associated with Mars?

(1) Curiosity

(2) Viking 1

(3) InSight

(4) Rosetta

Select the correct answer using the code given below:

A. 1 and 2 only

B. 2 and 4 only

C. 1, 2 and 3 only

D. 1, 2 and 4 only

Answer: C

Explanation:

Point (1) is correct: Curiosity is a car-sized rover designed to explore Gale Crater on Mars as part

of NASA’s Mars Science Laboratory mission (MSL). Curiosity was launched from Cape Canaveral

in 2011 and landed on mars in 2012. The rover is still operational.

Page 54: 11th May 2019 Live Test conducted by

Point (2) is correct: Viking 1 was the first of two spacecraft sent to Mars as part of NASA's Viking

program. On July 20, 1976, it became the second spacecraft to soft-land on Mars, and the first

soft lander to successfully perform its mission.

Point (3) is correct: InSight, short for Interior Exploration using Seismic Investigations, Geodesy

and Heat Transport, is a Mars lander. It is the first outer space robotic explorer to study in-

depth the "inner space" of Mars: its crust, mantle, and core. InSight landed on Mars on

November 26, 2018. It will studying the planet’s interior by measuring its heat output and

listening for marsquakes, which are seismic events similar to earthquakes on Earth.

Point (4) is incorrect: Rosetta was a space probe built by the European Space Agency launched

on 2 March 2004. Along with Philae, its lander module, Rosetta performed a detailed study of

comet 67P/Churyumov–Gerasimenko. Hence, option C. is the correct answer.

Source: http://vajiramias.com/current-affairs/mushrooms-on-

mars/5ca1cde91d5def0603edcbcb/

Q52. Which of the following is the main objective of “EMISAT”, recently launched by the Indian

Space Research Organization (ISRO)?

A. It is useful for remote asset management and aircraft tracking.

B. It is related to the studies of the cutting edge Internet-of-Things technology.

C. It will be used for digital audio and video broadcasting.

D. It is intended for electromagnetic spectrum measurement.

Answer: D

Explanation:

EMISAT satellite was successfully placed in its intended sun-synchronous polar orbit of 748 km

height in April, 2019. The satellite is intended for electromagnetic spectrum measurement. Its

devices have been developed by Defence Research and Development Organization (DRDO).

Hence statement D. is correct.

Source: http://vajiramias.com/current-affairs/pslv-c45-emisat/5ca1bce11d5def060080b9de/

Q53. With reference to the Renewable Energy Certificates (REC), consider the following

statements:

(1) They have been introduced in India as part of International Solar Alliance in 2015.

Page 55: 11th May 2019 Live Test conducted by

(2) Indian power bourses Indian Energy Exchange (IEX) and Power Exchange of India (PXIL) are

engaged in trading of RECs.

Which of the statements given above is/are correct?

A. 1 only

B. 2 only

C. Both 1 and 2

D. Neither 1 nor 2

Answer: B

Explanation:

Statement (1) is incorrect: Renewable Energy Certificates (REC) Mechanism,a market based

instrument, has been introduced in India in 2010. The REC mechanism is a market based

instrument to promote renewable sources of energy and development of market in electricity.

It provides an alternative voluntary route to a generator to sell his electricity from renewable

sources just like conventional electricity and offer the green attribute (RECs) separately to

obligated entities to fulfill their renewable purchase obligation (RPO). Under the RPO, bulk

purchasers like discoms, open access consumers and capacitive users are required to buy

certain proportion of renewable energy or RECs. They can buy RECs from renewable energy

producers to meet the RPO norms.

Statement (2) is correct: Indian Energy Exchange (IEX) and Power Exchange of India (PXIL) are

the two power bourses in the country, which are engaged in trading of RECs and electricity. The

trading of renewable energy certificates (RECs) is conducted on the last Wednesday of every

month.

Source: http://vajiramias.com/current-affairs/renewable-energy-certificates-

rec/5ca1f21a1d5def05fd1bc8e8/

Q54. With respect to “Bharat stage VI”, recently seen in news, consider the following

statements:

(1) The Bharat Stage emission standards and the timeline for implementation of the same are

set by the Environment Pollution (Prevention & Control) Authority of India.

(2) Chennai became the first city in the country to shift from Bharat stage-IV grade petrol and

diesel to Bharat stage-VI fuels.

Page 56: 11th May 2019 Live Test conducted by

Which of the statements given above is/are correct?

A. 1 only

B. 2 only

C. Both 1 and 2

D. Neither 1 nor 2

Answer: D

Explanation:

The Bharat Stage emission standards are standards instituted by the government to regulate

the output of air pollutants from motor vehicles from internal combustion engine equipment,

including motor vehicles. The BS IV norms had been enforced across the country since April

2017. In 2016, the Centre had announced that the country would skip the BS-V norms

altogether and adopt BS-VI norms by 2020. Implementation of the intermediate BS-V standard

was originally scheduled for 2019.

Statement (1) is incorrect: The standards and the timeline for implementation are set by

the Central Pollution Control Board under the Ministry of Environment & Forests and Climate

Change.

The main difference between BS IV and the BS VI is in the amount of Sulphur in the fuel which is

reduced from 50 ppm in BS IV fuel to 10 ppm in BS VI fuel for both gasoline and diesel. BS VI

norms will address one of the inherent flaws in the European emission standards which permits

diesel cars to emit more particulate matter and nitrogen oxide. NOx emissions from diesel cars

are expected to come down by nearly 70% and, from cars with petrol engines, by 25%.

Statement (2) is incorrect: In April 2018, Delhi became the first city in the country to shift from

Bharat stage-IV grade petrol and diesel to Bharat stage-VI fuels.

Page 57: 11th May 2019 Live Test conducted by

Source: http://vajiramias.com/current-affairs/bharat-stage-vi/5ca339b01d5def51862ca891/

Q55. Which of the following are the Manganese Ore producing states in India?

(1) Odisha

(2) Madhya Pradesh

(3) Karnataka

(4) Nagaland

(5) Maharashtra

Select the correct answer using the code given below:

A. 1 and 2 only

B. 1, 2 and 3 only

C. 2, 3, 4 and 5 only

D. 1, 2, 3 and 5 only

Answer: D

Explanation:

Indian manganese ore deposits occur mainly as metamorphosed bedded sedimentary deposits

associated with Gondite Series (Archaeans) of Madhya Pradesh (Balaghat, Chhindwara &

Jhabua districts), Maharashtra (Bhandara & Nagpur districts), Gujarat (Panchmahal district),

Page 58: 11th May 2019 Live Test conducted by

Odisha (Sundergarh district) and with Kodurite Series (Archaeans) of Odisha (Ganjam & Koraput

districts) and Andhra Pradesh (Srikakulam & Visakhapatnam districts).

State wise, Odisha tops the total reserves/resources with 44% share followed by Karnataka

22%, Madhya Pradesh 12%, Maharashtra & Goa 7% each, Andhra Pradesh 4% and Jharkhand

2%. Rajasthan, Gujarat, Telangana and West Bengal together shared the remaining about 2%

resources. Hence, option D. is the correct answer.

Balaghat Mine is the largest mine of the Manganese Ore India Ltd (MOIL) which produces one

of the best quality of manganese ore in the country and is also the deepest underground

manganese mine in Asia.

Manganese in alloy form is an essential input in steel making and is one of the most important

metals in an industrial economy, where it is used both in the ore form as such and as ferro-

manganese. Manganese improves strength, toughness, hardness and workability of steel.

Manganese dioxide is used for manufacturing dry cell batteries in which it functions as a

depolariser of hydrogen. In chemical industry, generally high-grade material is used for

potassium permanganate. Manganese ores of major commercial importance are: pyrolusite,

psilomelane, Manganite and braunite.

Source: http://vajiramias.com/current-affairs/steel-industry/5cb303421d5def05e9b6f10b/

Page 59: 11th May 2019 Live Test conducted by

Q56. Which of the following statement best describes the term “Kessler Syndrome”, recently

seen in news?

A. It is a scenario in which the amount of space debris in low earth orbit (LEO) is large enough to

cause space collisions to happen often.

B. It is a cutaneous disorder of hyperpigmentation characterized by reticulated pigmentation of

the person's skin.

C. It is a condition experienced by as many as half of all space travellers during their adaptation

to weightlessness once in orbit.

D. None of the above.

Answer: A

Explanation:

The Kessler syndrome is also called as collisional cascading or ablation cascade. It was proposed

by the NASA scientist Donald J. Kessler in 1978. It is a scenario in which the amount of space

debris in low earth orbit (LEO) is large enough to cause space collisions to happen often. Hence

option A. is the correct answer.

Space debris encompasses both natural (meteoroid) and artificial (man-made) particles.

Artificial debris (also referred as orbital debris) includes non-functional spacecraft, abandoned

launch vehicle stages, mission-related debris and fragmentation debris. These pieces move at

very great speeds, thus a collision with any other satellite in space could render that satellite

dysfunctional, if not destroy it. If an object is assessed to have the potential to hit International

Space Station (ISS), then ISS navigates away from its normal trajectory which is called as Debris

Avoidance Manoeuvres (DAMs).

Page 60: 11th May 2019 Live Test conducted by

Source: http://vajiramias.com/current-affairs/space-debris/5ca4896e1d5def54bc516d88/

Q57. With respect to “Purchasing Managers’ Index (PMI)”, consider the following statements:

(1) It is an indicator of business activity related to manufacturing sector only.

(2) It is published by NITI Aayog at the start of every month.

Which of the statements given above is/are correct?

A. 1 only

B. 2 only

C. Both 1 and 2

D. Neither 1 nor 2

Answer: D

Explanation:

Recently the Nikkei India Manufacturing Purchasing Managers’ Index (PMI) has slipped from

54.3 in February to 52.6 in March. Thus India’s manufacturing activities slowed down to a six-

month low in March, reflecting a loss of “growth momentum”.

Statement (1) is incorrect: It is an indicator of business activity -- both in the manufacturing and

services sectors. The headline PMI is a number from 0 to 100. A PMI above 50 represents an

Page 61: 11th May 2019 Live Test conducted by

expansion when compared to the previous month. A PMI reading under 50 represents a

contraction, and a reading at 50 indicates no change. The PMI is usually released at the start of

the month.

Statement (2) is incorrect: PMI was originally developed in 1948 by the US-based non-profit

group namely Institute of Supply Management (ISM). Now it is published in a variety of

different places, depending on the company and country. For India, the PMI Data is published

by Japanese firm Nikkei but compiled and constructed by Markit Economics.

Source: http://vajiramias.com/current-affairs/purchasing-managers-index-

pmi/5ca492d41d5def51862cf411/

Q58. Which of the following pairs are correctly matched?

Select the correct answer using the code given below:

A. 1 and 2 only

B. 1 and 3 only

C. 2 and 3 only

D. 1, 2 and 3

Answer: A

Explanation:

Pair (1) is correctly matched: Lord Howe Island is a crescent-shaped volcanic remnant in the

Tasman Sea between Australia and New Zealand. It is part of the state of New South Wales.

UNESCO records the Lord Howe Island Group as a World Heritage Site of global natural

significance. Recently the corals off Lord Howe Island, the world’s southernmost coral reef have

been hit by bleaching.

Page 62: 11th May 2019 Live Test conducted by

Pair (2) is correctly matched:Berdyansk is a port city in the Zaporizhia Oblast of south-east

Ukraine. It is on the northern coast of the Sea of Azov, which is the northern extension of the

Black Sea. Recently Ukraine is planning to build a naval base at Berdyansk on the Sea of Azov.

Pair (3) is not correctly matched: Tashigang is a village near an ancient monastery in the state of

Himachal Pradesh, India. It is the highest settlement in Spiti Valley, and is located in the Sutlej

river valley near the India-Tibet border. The world's highest polling station is now in the sleepy

hamlet of Tashigang in the Buddhist-dominated Lahaul-Spiti district of Himachal Pradesh at an

altitude of 15,256 feet. Earlier the tiny settlement of Hikkim in the same district located at an

altitude of about 14,400 feet was the highest polling station in the country.

Source: http://vajiramias.com/current-affairs/corals-off-lord-howe-

island/5ca5e0571d5def51890ec3a3/

Q59. The term “Transfer Pricing” was in news recently. Which one of the following statements

provides its most apt description?

A. It is that price of a commodity or a security at which the market clears a commodity or a

security.

B. It is the floor price that must be paid irrespective of the market price.

C. It is a tariff or tax imposed on goods that are transported across international borders.

D. It is the rates/prices that are used when selling goods or services between company divisions

and departments, or between a parent company and a subsidiary.

Answer: D

Explanation:

Page 63: 11th May 2019 Live Test conducted by

Transfer pricing is the rates/prices that are used when selling goods or services between

company divisions and departments, or between a parent company and a subsidiary. In

principle a transfer price should match either what the seller would charge an

independent, arm’s length customer, or what the buyer would pay an independent, arm’s

length supplier. However, they have become a major tool for tax avoidance. i.e. they are

misused to for lower profits in a division of an enterprise that is located in a country that levies

high taxes (less profit, less tax payed) and raise profits in a country that is a tax haven that

levies no or low taxes. Hence, option D. is the correct answer.

Value Addition

Clearing price- It is that price of a commodity or a security at which the market clears a

commodity or a security. Quantity supplied is equal to quantity demanded and buyers and

sellers conduct the trade.

Fair Trade Price- In the commodities market, fair trade price is the minimum price that

importers must pay to the producers of some agricultural products such as coffee and banana.

It is the floor price that must be paid irrespective of the market price.

Source: http://vajiramias.com/current-affairs/advance-pricing-agreement-

apas/5ca5b73a1d5def51862d2a76/

Q60. Recently the healthcare facilities in several countries have reported that the Candida auris

has been causing severe illness in hospitalized patients. In this context, what is Candida auris?

A. Virus

B. Bacteria

C. Fungus

D. Protozoa

Answer: C

Explanation:

Candida auris (also called C. auris) is a fungus. It can cause bloodstream infections, wound

infections, ear infections and even death, particularly in hospital and nursing home patients

with serious medical problems. Auris is the Latin word for ear. Despite its name, C. auris can

also affect many other regions of the body and can cause invasive infections, including

bloodstream infections and wound infections. Although C. auris was first identified in 2009 in

Japan, it has spread quickly and caused infections in more than a dozen countries. Antifungal

Page 64: 11th May 2019 Live Test conducted by

medicines commonly used to treat Candida infections often don’t work for Candida auris.

Some C. auris infections have been resistant to all three types of antifungal medicines. Hence,

option C. is the correct answer.

Source: http://vajiramias.com/current-affairs/candida-auris/5cab1b991d5def5184e7d7b6/

Q61. With respect to “Liquidity Coverage Ratio”, consider the following statements:

(1) It indicates the proportion of highly liquid assets held by banks to ensure their ability to

meet short-term obligations.

(2) It is calculated by dividing a bank’s high-quality liquid assets by its total net cash flows, over

a 30-day stress period.

Which of the statements given above is/are correct?

A. 1 only

B. 2 only

C. Both 1 and 2

D. Neither 1 nor 2

Answer: C

Explanation:

Statement (1) is correct:Liquidity Coverage Ratio (LCR) indicates the proportion of highly liquid

assets held by banks to ensure their ability to meet short-term obligations. This ratio is

essentially a generic stress test that aims to anticipate market-wide shocks and make sure that

financial institutions possess suitable capital preservation, to ride out any short-term liquidity

disruptions that may plague the market.

Statement (2) is correct:The LCR is calculated by dividing a bank’s high-quality liquid assets by

its total net cash flows, over a 30-day stress period. The high-quality liquid assets include only

those with a high potential to be converted easily and quickly into cash. The three categories of

liquid assets with decreasing levels of quality are level 1, level 2A, and level 2B.

Source: http://vajiramias.com/current-affairs/liquidity-coverage-ratio-

lcr/5ca727eb1d5def54bc51e683/

Q62. Which of the following are the Rabi crops in India?

(1) Chickpea

Page 65: 11th May 2019 Live Test conducted by

(2) Wheat

(3) Bajra

(4) Sorghum

Select the correct answer using the code given below:

A. 1 and 4 only

B. 1 and 2 only

C. 1, 2 and 3 only

D. 1, 2, 3 and 4

Answer: B

Explanation:

Chickpea is a cool season crop. In India, chickpea is generally sown in September-October and

harvested in January-February. More than 90% of chickpea cultivation area is in South Asia,

including India. Chickpea originated in the Mediterranean/south-west Asia and migrated to

south Asia. Globally, more than 70% yield is lost due to drought and increasing temperatures.

The agricultural crop year in India is from July to June. The Indian cropping season is classified

into two main seasons-(i) Kharif and (ii) Rabi based on the monsoon. The kharif cropping season

is from July –October during the south-west monsoon and the Rabi cropping season is from

October-March (winter). The crops grown between March and June are summer crops. Pakistan

and Bangladesh are two other countries that are using the term ‘kharif’ and ‘rabi’ to describe

about their cropping patterns. The terms ‘kharif’ and ‘rabi’ originate from Arabic language

where Kharif means autumn and Rabi means spring.

The kharif crops include rice, maize, sorghum, pearl millet/bajra, finger millet/ragi (cereals),

arhar (pulses), soyabean, groundnut (oilseeds), cotton etc. The rabi crops include wheat, barley,

oats (cereals), chickpea/gram (pulses), linseed, mustard (oilseeds) etc. Hence, option B. is the

correct answer.

Recently an international team led by the Hyderabad-based ICRISAT has identified in chickpea

four important genes for heat tolerance and three important genes for drought tolerance.

Source: http://vajiramias.com/current-affairs/chickpea/5cc80b911d5def7e238a29e3/

Q63. Which of the following two Organisations together constitute the World Bank?

Page 66: 11th May 2019 Live Test conducted by

(1) The International Bank for Reconstruction and Development (IBRD)

(2) The International Development Association (IDA)

(3) The International Finance Corporation (IFC)

(4) The International Monetary Fund (IMF)

Select the correct answer using the code given below:

A. 1 and 2 only

B. 1 and 3 only

C. 2 and 3 only

D. 1 and 4 only

Answer: A

Explanation:

Senior US Treasury official David Malpass has been selected as the 13 th President of the World

Bank for a five-year term. The President of the Bank is the president of the entire World Bank

Group. Traditionally, the U.S., which is the World Bank’s largest investor, normally picks its

president, with the Europeans picking the International Monetary Fund head.

The World Bank is an international financial institution that provides loans to countries of the

world for capital projects. The World Bank is a component of the World Bank Group. The World

Bank Group consists of five organizations – (i) The International Bank for Reconstruction and

Development (IBRD) (ii) The International Development Association (IDA) (iii) The International

Finance Corporation (IFC) (iv)The Multilateral Investment Guarantee Agency (MIGA) (v) The

International Centre for Settlement of Investment Disputes (ICSID).

Together, IBRD and IDA form the World Bank, which provides financing, policy advice, and

technical assistance to governments of developing countries. IDA focuses on the world’s

poorest countries, while IBRD assists middle-income and creditworthy poorer

countries. Hence, option A. is the correct answer.

Source: http://vajiramias.com/current-affairs/david-malpass/5ca87a4f1d5def51890f3afe/

Q64. A group of jumping spiders (Habrocestum longispinum) that mostly occur in Eurasia and

Africa, has been recently spotted for the first time in which of the following states in India?

A. Arunachal Pradesh

Page 67: 11th May 2019 Live Test conducted by

B. Gujarat

C. Odisha

D. Kerala

Answer: D

Explanation:

A group of jumping spiders that mostly occurs in Eurasia and Africa has been spotted for the

first time in Ernakulam’s Illithodu forests. The spider has a single long spine on the underside of

both its first legs, and this gave it its scientific name Habrocestum longispinum (after Latin

‘longe’ meaning long and ‘spinae’ for spine). These spiders belong to the genus Habrocestum

that has been recorded mostly in Eurasia and Africa and never in India, till now. These groups of

jumping spiders are predominantly brownish-black in colour with white and creamy-yellow

patches. The study extends the range of these spiders to India. The discovery also lends support

to the continental drift theory that suggests that the world’s continents were one large,

contiguous landmass where these creatures thrived many millions of years ago. Hence option

D. is the correct answer.

Source: http://vajiramias.com/current-affairs/habrocestum-

longispinum/5ca884851d5def518a8b490a/

Q65. Which of the following pairs is/are correctly matched?

Select the correct answer using the code given below:

A. 1 only

B. 2 and 3 only

C. 1 and 2 only

D. 1, 2 and 3

Page 68: 11th May 2019 Live Test conducted by

Answer: C

Explanation:

Recently around 4,700 Konyak Naga women came together in an attempt to set a Guinness

World Record for the “Largest Traditional Konyak Dance”.

Pair (1) is correctly matched: Konyak is one of the 16 Naga tribes and people of this community

live mainly in the Mon district of Nagaland. They are easily distinguishable from other Naga

tribes by their pierced ears; and tattoos which they have all over their faces, hands, chests,

arms, and calves. Aoleng, a festival celebrated in the first week of April to welcome the spring,

is the biggest festival of the Konyaks. Another festival, ‘Lao Ong Mo’ is the traditional harvest

festival celebrated in the months of August/September.

Pair (2) is correctly matched: Bhavai is a popular form of folk theatre in Gujarat. It is a

performance where a story is told through an act with some music. The word “bhavai” means

expression. Bhavai is a basically a troupe drama. The chief of the troupe known as ‘Nayak’ who

commences the drama by worshiping the Goddess Amba.

Pair (3) is not correctly matched: Togalu Gombeyaata is a puppet show unique to the state of

Karnataka, India. Togalu Gombeyaata translates to a play of leather dolls in the native language

of Kannada. It is a form of shadow puppetry.

Source: http://vajiramias.com/current-affairs/konyak-dance/5ca883421d5def5184e74a18/

Q66. Menhirs, Dolmens and Cromlechs are:

A. Isolated residual hills

B. U-shaped valleys shaped by the glaciers

C. Ancient rock structures

D. Crescent shaped dunes in deserts

Answer: C

Explanation:

Dolmens, menhirs, and cromlechs are mysterious rock structures spread throughout the world.

Dolmens are large, stone structures, placed horizontally on top of a stone base, or a closed

stone box with a circular (most often), triangular, or square entrance. Menhirs are stone pillars

placed vertically on the ground. Cromlechs are several elongated stones places in the ground

Page 69: 11th May 2019 Live Test conducted by

(menhirs), forming one or several concentric circles. And sometimes, in the center of such a

structure, there is another stone – menhir or dolmen. Hence, option C. is the correct answer.

Recently, Indian archaeologists discovered pictographs etched on large stone slabs, menhirs —

large standing stones — and a necropolis — a large cemetery — among other artefacts from

Vangchhia village. It lies in the Champhai-Farkawn mountain range of Mizoram near Indo-

Myanmar border. The excavation at Vangchhia exposed a good number of menhirs, Mizo

traditional and megalithic burial structures and one of the most profound discoveries is the

water pavilion that comprises multiple circular and square holes. In another unique

find, remnants of small man-made cave dwellings, topped with menhirs taller than an average

full-grown man, have been unearthed at the hilltop village of Dungtlang.

Source: https://www.thehindu.com/news/national/decoding-vangchhias-ancient-art-of-

holding-water-in-rock-amid-mizorams-hills/article26225455.ece

https://indianexpress.com/article/lifestyle/destination-of-the-week/the-discovery-of-

vangchhia/

http://vajiramias.com/current-affairs/domkhar-rock-art-

sanctuary/5ca9ce601d5def51862df48e/

Q67. Consider the following statements, with reference to the Monetary Policy:

(1) If Repo rate is increased then it means banks have to borrow money at high cost.

(2) The interest rate for Marginal standing facility (MSF) borrowing was originally set at one

percent higher than the repo rate.

Page 70: 11th May 2019 Live Test conducted by

(3) Reverse repo rate is the rate of interest offered by RBI, when banks deposit surplus funds

with the RBI for short periods.

Which of the statements given above is/are correct?

A. 1 only

B. 2 and 3 only

C. 1 and 3 only

D. 1, 2 and 3

Answer: D

Explanation:

Recently, Monetary Policy Committee of the RBI has cut the policy repo rate by 25 basis points

to 6.0 per cent from 6.25 per cent. Consequently, the reverse repo rate under the LAF stands

adjusted to 5.75 %, and the marginal standing facility (MSF) rate and the Bank Rate to 6.25 %.

Statement (1) is correct:Repurchase rate (or Repo Rate) is the rate at which the RBI lends

money to commercial banks in case banks are facing shortfall of funds. If Repo rate is increased

then it means banks have to borrow money at high cost.

Statement (2) is correct: Marginal standing facility (MSF) is a window for banks to borrow from

the Reserve Bank of India in an emergency situation when inter-bank liquidity dries up

completely. Banks borrow from the central bank by pledging government securities at a rate

higher than the repo rate under liquidity adjustment facility or LAF in short. The MSF rate is

pegged 100 basis points or a percentage point above the repo rate. Under MSF, banks can

borrow funds up to one percentage of their net demand and time liabilities (NDTL).

Statement (3) is correct: Reverse repo rate is the rate of interest offered by RBI, when banks

deposit surplus funds with the RBI for short periods. In the recent monetary policy review, it

has been adjusted to 6.25 %. Hence, option D. is the correct answer.

Source: http://vajiramias.com/current-affairs/monetary-policy-

statement/5ca7273e1d5def5184e70e9f/

Q68. Who among the following groups are eligible for ‘Proxy Voting’?

(1) Armed Forces

(2) Boarder Road Organisation

Page 71: 11th May 2019 Live Test conducted by

(3) Non-resident Indians

(4) Assam Rifles

Select the correct answer using the code given below:

A. 1 and 4 only

B. 1 and 3 only

C. 1, 2 and 4 only

D. 1, 2, 3 and 4

Answer: C

Explanation:

According to the Election Commission of India (ECI), in the 2019 Indian General Election, service

Voters are going to play a crucial and decisive role in cases of close contests. Service voter are

those who have service qualification. They include member of Armed Forces of the Union of

India, members of Armed Police forces of the States serving outside that state and persons

employed under Government of India on posts outside India. Service Voter can cast his/her

vote by proxy also. They may appoint (By applying to returning officer in Form 13 F of ECI) any

person as his/her proxy to cast vote on his/her behalf in his/her at the polling station. Proxy

should be a registered voter of that constituency.

Eligible Organizations: As per existing arrangements, members of India Army, Navy and Air

force, Boarder Road Organisation, BSF, ITBP, Assam Rifles, NSG, CRPF, CISF and SSB are eligible

to be registered as service voters.

The key bill to allow proxy voting rights to overseas Indians and make electoral law gender

neutral for service voters was passed by Lok Sabha in August, 2018 and has been awaiting Rajya

Sabha nod. Hence, option C. is the correct answer.

Source: http://vajiramias.com/current-affairs/service-voters/5cac5d451d5def54bc52f94b/

Q69. With reference to the Small Finance Bank (SFB), consider the following statements:

(1) SFBs have the status of a scheduled bank under the RBI Act, 1934.

(2) It can undertake financial services like distribution of mutual fund units, insurance products

and pension products without prior approval from the RBI.

Which of the statements given above is/are correct?

Page 72: 11th May 2019 Live Test conducted by

A. 1 only

B. 2 only

C. Both 1 and 2

D. Neither 1 nor 2

Answer: A

Explanation:

Small Finance Bank (SFB) are a niche type of banks in India who cater to mainly customers not

being serviced by the big commercial banks such as small business units, small farmers, MSMEs

and various other unorganised sectors.

Statement (1) is correct: SFBs have the status of a scheduled bank under Section 42(6)A. of the

RBI Act, 1934. Like commercial banks, they provide both the saving and lending facility.

Statement (2) is incorrect: Small banks can undertake financial services like distribution of

mutual fund units, insurance products, pension products, and so on, but not without prior

approval from the RBI.

Source: http://vajiramias.com/current-affairs/small-finance-

banks/5cab19e71d5def5184e7d784/

Q70. With respect to “Atal Ranking of Institutions on Innovation Achievements (ARIIA)”,

consider the following statements:

(1) It is an initiative of the NITI Aayog.

(2) Its objective is to rank all major higher educational institutions and universities in India on

indicators related to innovation.

Which of the statements given above is/are correct?

A. 1 only

B. 2 only

C. Both 1 and 2

D. Neither 1 nor 2

Answer: B

Page 73: 11th May 2019 Live Test conducted by

Explanation:

Statement (1) is incorrect: Atal Ranking of Institutions on Innovation Achievements (ARIIA) is an

initiative of innovation cell of Ministry of Human Resource Development (MHRD).

Statement (2) is correct:Its objective is to rank all major higher educational institutions and

universities in India on indicators related to “Innovation and Entrepreneurship Development”

amongst students and faculties.

IIT Madras emerged as the “Top innovative Institution” in the country in the maiden edition of

the Atal Ranking of Institutions on Innovation Achievements (ARIIA) released by the President

of India.

Source: http://vajiramias.com/current-affairs/atal-ranking-of-institutions-on-innovation-

achievements-ariia/5cac5e2e1d5def5184e81f60/

Q71. The National Green Tribunal has jurisdiction on which of the following laws?

(1) The Biological Diversity Act, 2002

(2) The Environment (Protection) Act, 1986

(3) The Wildlife (Protection) Act, 1972

(4) The Forest (Conservation) Act, 1980

Select the correct answer using the code given below:

A. 1 and 2 only

B. 2 and 3 only

C. 1, 2 and 4 only

D. 1, 2 and 3 only

Answer: C

Explanation:

National Green Tribunal is a statutory body established under the National Green Tribunal Act

2010. The Tribunal shall not be bound by the procedure laid down under the Code of Civil

Procedure, 1908, but shall be guided by principles of natural justice. The Tribunal should

dispose applications or appeals finally within 6 months of filing of the same.

Page 74: 11th May 2019 Live Test conducted by

The NGT has the power to hear all civil cases relating to environmental issues and questions

that are linked to the implementation of laws listed in Schedule I of the NGT Act. These include

the following:

(i) The Water (Prevention and Control of Pollution) Act, 1974;

(ii) The Water (Prevention and Control of Pollution) Cess Act, 1977;

(iii) The Forest (Conservation) Act, 1980;

(iv) The Air (Prevention and Control of Pollution) Act, 1981;

(v) The Environment (Protection) Act, 1986;

(vi) The Public Liability Insurance Act, 1991;

(vii) The Biological Diversity Act, 2002.

This means that any violations pertaining only to these laws, or any order / decision taken by

the Government under these laws can be challenged before the NGT. Importantly, the NGT has

not been vested with powers to hear any matter relating to the Wildlife (Protection) Act, 1972,

the Indian Forest Act, 1927 and various laws enacted by States relating to forests, tree

preservation etc. Therefore, specific and substantial issues related to these laws cannot be

raised before the NGT.

Recently the National Green Tribunal (NGT) has formed a committee to look into Yamuna

floodplain erosion in Sonipat.

Source: http://vajiramias.com/current-affairs/committee-on-yamuna-floodplain-erosion-in-

sonipat/5cac5c2a1d5def518910158d/

Q72. With respect to “Press Council of India (PCI)”, consider the following statements:

(1) It is headed by the retired Chief Justice of India as per the Press Council Act, 1978.

(2) It adjudicates the complaints against and by the press for violation of ethics and for violation

of the freedom of the press respectively.

Which of the statements given above is/are correct?

A. 1 only

B. 2 only

C. Both 1 and 2

Page 75: 11th May 2019 Live Test conducted by

D. Neither 1 nor 2

Answer: B

Explanation:

Statement (2) is correct: The Press Council of India (PCI) is a statutory body functioning under

the Press Council Act, 1978. It adjudicates the complaints against and by the press for violation

of ethics and for violation of the freedom of the press respectively.

Statement (1) is incorrect: It is headed by a Chairman, who has by convention, been a retired

judge of the Supreme Court of India.

The Council consists of 28 other members of -whom 20 represent the press and are nominated

by the press organisations/news agencies, 5 members are nominated from the two Houses of

Parliament and 3 represent cultural, literary and legal fields as nominees of the Sahitya

Academy, University Grants Commission and the Bar Council of India. The members serve on

the Council for a term of three years. A retiring member shall be eligible for re-nomination for

not more than one term.

Source: http://vajiramias.com/current-affairs/media-and-

elections/5cadb1ed1d5def4bd26eef97/

Q73. With reference to the Serious Fraud Investigation Office (SFIO), consider the following

statements:

(1) It is a statutory body setup in 2003 on the basis of the recommendations of the Naresh

Chandra Committee.

(2) SFIO is under the Union Ministry of Finance.

Which of the statements given above is/are correct?

A. 1 only

B. 2 only

C. Both 1 and 2

D. Neither 1 nor 2

Answer: A

Explanation:

Page 76: 11th May 2019 Live Test conducted by

Statement (1) is correct: The Serious Fraud Investigation Office (SFIO) was setup in 2003 on the

basis of the recommendations of the Naresh Chandra Committee and in the backdrop of stock

market scams. It is a statutory body under the Section 211 of the Companies Act, 2013.

It is a multi-disciplinary organization consisting of experts in the field of accountancy, forensic

auditing, law, information technology, investigation, company law, capital market and taxation

for detecting and prosecuting or recommending for prosecution white- collar crimes/frauds.

Statement (2) is incorrect: It is under the Union Ministry of Corporate Affairs. SFIO has head

office in New Delhi and regional offices in Maharashtra, Andhra Pradesh, Tamil Nadu and West

Bengal.

Source: http://vajiramias.com/current-affairs/serious-fraud-investigation-office-

sfio/5ca878ab1d5def54bc521f3e/

Q74. Which of the following are the tobacco producing states in India?

(1) Andhra Pradesh

(2) Karnataka

(3) Gujarat

(4) Kerala

Select the correct answer using the code given below:

A. 1 and 3 only

B. 1 and 4 only

C. 2 and 3 only

D. 1, 2 and 3 only

Answer: D

Explanation:

Tobacco is grown in an area of about 4.5 lakh hectares over 15 states and according to the

agriculture ministry. Tobacco accounts for less than 0.3% of India’s arable land. Andhra

Pradesh (45%), Karnataka (26%), Gujarat (14%), Uttar Pradesh (5%), Tamil Nadu (2%), Bihar

(2%) and West Bengal (1%) are the major tobacco growing states. India produces 750 mn kg of

tobacco annually, making it the world’s second largest producer after China, which produces

2,300 mn kg. India also ranks second in tobacco exports (260 mn kg), after Brazil (270 mn kg).

Page 77: 11th May 2019 Live Test conducted by

The country accounts for about 6% by volume but less than 1% by value of the world tobacco

trade. However about 15% of the land under tobacco cultivation in India has been shifted to

other crops in the last three years under the government’s Crop Diversification Programme

(CDP). Hence, option D. is the correct answer.

According to the World Health Organisation (WHO), Global cigarette sales in 2017 stood at

$700 billion, which was 250 times more than what the WHO needed to protect human health.

In India, where the mean age at initiation to daily smoking is 18.7 years, the total tax revenue

collected from tobacco products is more than ₹34,000 crore annually.

Source: http://vajiramias.com/current-affairs/tobacco-abuse/5caf0e651d5def4bd54b0682/

Q75. With reference to the Event Horizon Telescope (EHT), consider the following statements:

(1) It is an array of eight ground-based radio telescopes around the globe forged through

international collaboration.

(2) It was designed to capture images of far side of the moon.

Which of the statements given above is/are correct?

A. 1 only

B. 2 only

C. Both 1 and 2

D. Neither 1 nor 2

Answer: A

Explanation:

Recently the first image of the black hole inside galaxy Messier 87 was published. The black

hole was given the name Pōwehi, meaning "embellished dark source of unending creation" in

Hawaiian.

Statement (1) is correct: The Event Horizon Telescope (EHT) is a planet-scale array of eight

ground-based radio telescopes forged through international collaboration. The EHT links

telescopes around the globe to form an Earth-sized virtual telescope with unprecedented

sensitivity and resolution.

Statement (2) is incorrect: The EHT was designed to capture images of a black hole. It offers

scientists a new way to study the most extreme objects in the Universe predicted by Einstein’s

Page 78: 11th May 2019 Live Test conducted by

general relativity during the centennial year of the historic experiment that first confirmed the

theory.

Thirteen partner institutions worked together to create the EHT. Key funding was provided by

the US National Science Foundation (NSF), the EU’s European Research Council (ERC), and

funding agencies in East Asia. The EHT observations use a technique called very-long-baseline

interferometry (VLBI).

Source: http://vajiramias.com/current-affairs/event-horizon-

telescope/5caf097c1d5def4bd4db6e35/

Q76. What is ‘Honeypot’, recently seen in the news?

A. It is the new heavy thermonuclear intercontinental ballistic missile (ICBM) developed by

Russia.

B. It is a system designed to mimic likely targets of cyber-attackers.

C. It is a traditional beekeeping technology.

D. None of the above

Answer: B

Page 79: 11th May 2019 Live Test conducted by

Explanation:

A honeypot is a system designed to mimic likely targets of cyber-attackers, so that security

researchers can monitor cybercriminal behaviour. Types of honeypots are a low-interaction

honeypot is a honeypot that, once found by the hacker, will not be of much use to them and a

high-interaction honeypot permits the attacker to go further in order to gather additional

information about their intentions. Hence, option B. is the correct answer.

Source: http://vajiramias.com/current-affairs/honeypots/5caef41a1d5def4bd7bfee91/

Q77. Arrange the following peaks from West to East:

(1) Mount Everest

(2) Kanchenjunga

(3) Nanda Devi

(4) Kamet Peak

Select the correct answer using the code given below:

A. 3-1-4-2

B. 3-4-1-2

C. 1-3-2-4

D. 4-3-1-2

Answer: D

Explanation:

The correct sequence of peaks in the direction of West to East is-

Page 80: 11th May 2019 Live Test conducted by

Nepal has recently sent for the first time, its own team of experts to re-measure the height of

Mount Everest. Survey of India had measured the height as 8848 meter in 1954 and it has been

adopted as authentic height by the Government of Nepal till date.

Source: http://vajiramias.com/current-affairs/height-of-mount-

everest/5caef5861d5def4bd26f37fe/

Q78. Which of the following is/are the application(s) of Hyperspectral sensing?

(1) To identify specific plants in a highly vegetated area.

(2) To detect seams of valuable minerals.

Select the correct answer using the code given below:

A. 1 only

B. 2 only

C. Both 1 and 2

D. Neither 1 nor 2

Answer: C

Explanation:

Hyperspectral imaging, like other spectral imaging, collects and processes information from

across the electromagnetic spectrum. The goal of hyperspectral imaging is to obtain the

spectrum for each pixel in the image of a scene, with the purpose of finding objects, identifying

materials, or detecting processes.

Hyperspectral sensing, for example, could be used to detect seams of valuable minerals within

rock faces or to identify specific plants in a highly vegetated area. Spectroscopy can also be

used to detect individual absorption features due to specific chemical bonds in a solid, liquid, or

gas. Hence, option C. is the correct answer.

Source: http://vajiramias.com/current-affairs/tiny-

spectrometers/5caef3bf1d5def4bd7bfee80/

Q79. Which of the following pairs is/are correctly matched?

Page 81: 11th May 2019 Live Test conducted by

Select the correct answer using the code given below:

A. 1 only

B. 2 and 3 only

C. 1 and 3 only

D. 1, 2 and 3

Answer: D

Explanation:

Pair (1) is correctly matched: The Convention on Cybercrime, also known as the Budapest

Convention on Cybercrime or the Budapest Convention, is the first international treaty seeking

to address Internet and computer crime (cybercrime)by harmonizing national laws, improving

investigative techniques, and increasing cooperation among nations. This convention of the

council of Europe is the only binding international instrument on this issue that addresses

Internet and computer crime by harmonizing national laws, improving legal authorities for

investigative techniques, and increasing cooperation among nations. It deals with issues such as

infringements of copyright, computer-related fraud, child pornography and violations of

network security.

Pair (2) is correctly matched: The Rotterdam Convention (formally, the Rotterdam

Convention on the Prior Informed Consent Procedure for Certain Hazardous Chemicals and

Pesticides in International Trade) is a multilateral treaty to promote shared responsibilities in

relation to importation of hazardous chemicals. The Convention entered into force on 24

February 2004 and became legally binding for its Parties.

Pair (3) is correctly matched: Stockholm Convention on Persistent Organic Pollutants is an

international environmental treaty, signed in 2001 and effective from May 2004, that aims to

Page 82: 11th May 2019 Live Test conducted by

eliminate or restrict the production and use of persistent organic pollutants (POPs). POPs are

chemicals that remain intact in the environment for long periods, become widely distributed

geographically, accumulate in the fatty tissue of living organisms and are toxic to humans and

wildlife. POPs circulate globally and can cause damage wherever they travel.

Source: http://vajiramias.com/current-affairs/current-state-of-cybercrime-

2019/5cc6b15f1d5def07d098bcfb/

Q80. With respect to the “International Criminal Court (ICC)”, consider the following

statements:

(1) It is the primary judicial branch of the United Nations.

(2) All members of the United Nations are party to the ICC Statute.

Which of the statements given above is/are correct?

A. 1 only

B. 2 only

C. Both 1 and 2

D. Neither 1 nor 2

Answer: D

Explanation:

Statement (1) is incorrect: International Court of Justice (ICJ) is the primary judicial branch of

United Nations and settles legal disputes and provides advisory opinions submitted to it by its

member states. ICJ is a civil court. The International Criminal Court is an intergovernmental

organization and international tribunal that sits in The Hague in the Netherlands. The ICC has

the jurisdiction to prosecute individuals for the international crimes of genocide, crimes against

humanity, war crimes, and crimes of aggression.

Statement (2) is incorrect: It was set up after the ad hoc tribunals to deal with Rwanda war

crimes proved ineffective. ICC is a criminal court. But most nations are not a signatory to ICC

including India, China, USA and Russia.

ICC is based on the principle of complementarities. It was not created to supplant the authority

of national courts. It is funded by contribution from state parties to the Rome Statute;

voluntary contributions from the U.N; voluntary contributions from governments, international

organizations, individuals, corporations and other entities.

Page 83: 11th May 2019 Live Test conducted by

Source: http://vajiramias.com/current-affairs/omar-al-bashir/5cb0350a1d5def61398fb7d2/

Q81. With reference to the Pharmacovigilance Program of India, consider the following

statements:

(1) It was launched by Central Drugs Standard Control Organisation (CDSCO) under the aegis of

Ministry of Chemicals and Fertilizers.

(2) Its objective is to monitor the Adverse drug Reactions (ADRs) among Indian population.

Which of the statements given above is/are correct?

A. 1 only

B. 2 only

C. Both 1 and 2

D. Neither 1 nor 2

Answer: B

Explanation:

Statement (1) is incorrect: Pharmacovigilance Program of India was launched by Central Drugs

Standard Control Organisation (CDSCO) under the aegis of Ministry of Health & Family Welfare.

Statement (2) is correct: Its objective is to monitor the Adverse drug Reactions (ADRs) among

Indian population and helps the regulatory authority of India in taking decision for safe use of

medicines.

The National Co-ordination Centre of the Pharmacovigilance Programme of India (NCC-PvPI) is

the Union Health Ministry’s pharmaceutical watchdog. In 2017, NCC-PvPI was launched as a

WHO Collaborating Centre for Pharmacovigilance in Public Health Programmes and Regulatory

Services.

Page 84: 11th May 2019 Live Test conducted by

Source: http://vajiramias.com/current-affairs/pharmacovigilance-programme-of-india-

pvpi/5cb1a6841d5def0608abbb11/

Q82. Which of the following pairs is/are correctly matched?

Select the correct answer using the code given below:

A. 1 only

B. 2 and 3 only

C. 1 and 2 only

D. 1, 2 and 3

Answer: A

Page 85: 11th May 2019 Live Test conducted by

Explanation:

Pair (1) is correctly matched: MCPH1 is a Protein Coding gene that scientists believe plays a role

in the development of the human brain. Certain mutations in MCPH1, when homozygous, cause

primary microcephaly—a severely diminished brain. According to a study published in Beijing-

based National Science Review, Chinese scientists have implanted human brain genes into

monkeys, in a study on evolution of human intelligence.

Pair (2) is not correctly matched:VISIONS-2 Provides a Look at Earth’s Escaping Atmosphere.

The Visualizing Ion Outflow via Neutral Atom Sensing-2 or VISIONS-2 rockets is looking at

atmospheric escape, the process whereby Earth is slowly leaking its atmosphere into space.

VISIONS-2 flew two rockets into the northern polar cusp, where it used an imaging technique to

map oxygen outflow from the aurora. Using this technique, VISIONS-2 takes a different

approach from many other missions, which attempt to combine data from many outflow

events. Instead, VISIONS-2 hopes to acquire a great deal of data about a single oxygen outflow

event. Not all outflow events are the same, but understanding one in great detail would

provide significant scientific value. VISIONS-2 was the first of nine sounding rocket missions

launching over the next 14 months as part of the Grand Challenge Initiative (GCI).

Pair (3) is not correctly matched: Stuxnet is an extremely sophisticated computer worm that

exploits multiple previously unknown Windows zero-day vulnerabilities to infect computers and

spread. Its purpose was not just to infect PCs but to cause real-world physical effects.

Specifically, it targets centrifuges used to produce the enriched uranium that powers nuclear

weapons and reactors. It's now widely accepted that Stuxnet was created by the intelligence

agencies of the United States and Israel. The classified program to develop the worm was given

the code name "Operation Olympic Games"; it was begun under President George W. Bush and

continued under President Obama. While neither government has ever officially acknowledged

developing Stuxnet, it is believed to be responsible for causing substantial damage to Iran's

nuclear program.

Source: http://vajiramias.com/current-affairs/mcph1/5cb19a7e1d5def060bd25e64/

Q83. With reference to the Great Backyard Bird Count (GBBC) 2019, consider the following

statements:

(1) In India, GBBC is coordinated by the Bombay Natural History Society (BNHS).

(2) Tamil Nadu's Salem district submitted the largest number of checklists from India.

Which of the statements given above is/are correct?

A. 1 only

Page 86: 11th May 2019 Live Test conducted by

B. 2 only

C. Both 1 and 2

D. Neither 1 nor 2

Answer: B

Explanation:

The Great Backyard Bird Count (GBBC) is an event to create a real-time snapshot of bird

populations. Participants are asked to count birds for as little as 15 minutes (or as long as they

wish) on one or more days of the four-day event and report their sightings online at

birdcount.org. Anyone can take part in the Great Backyard Bird Count, from beginning bird

watchers to experts. One can participate from his/her backyard, or anywhere in the world. Each

checklist submitted during the GBBC helps researchers at the Cornell Lab of Ornithology and

the National Audubon Society learn more about how birds are distributed across the country

and how they are affected by changes in habitat and weather.

Statement (1) is incorrect: Indian birders have participated in the GBBC since 2013. In India,

GBBC is coordinated by the Bird Count India (BCI) collective, a coming-together of a number of

groups and organisations that are interested in birds, nature and conservation.

Statement (2) is correct: India contributed 22,273 lists, the second highest after the U.S. Tamil

Nadu’s Salem district submitted the largest number of checklists, at 8,420, among all

participating districts to the global Great Backyard Bird Count 2019.

Source: http://vajiramias.com/current-affairs/great-backyard-bird-count-gbbc-

2019/5cb2e5551d5def05e63e6bca/

Q84. Mediterranean Sea borders which among the following countries?

(1) Israel

(2) Turkey

(3) Italy

(4) Russia

Select the correct answer using the code given below:

A. 1 and 4 only

B. 1 and 2 only

Page 87: 11th May 2019 Live Test conducted by

C. 2 and 3 only

D. 1, 2 and 3 only

Answer: D

Explanation:

The countries surrounding the Mediterranean in clockwise order are Spain, France, Monaco,

Italy, Slovenia, Croatia, Bosnia and Herzegovina, Montenegro, Albania, Greece, Turkey, Syria,

Lebanon, Israel, Egypt, Libya, Tunisia, Algeria, and Morocco; Malta and Cyprus are island

countries in the sea.

A new government for the West Bank-based Palestinian Authority led by PM Mohammed

Ishtayeh has been sworn-in recently. Until 1948, Palestine typically referred to the geographic

region located between the Mediterranean Sea and the Jordan River. Arab people who call this

territory home are known as Palestinians.

Source: http://vajiramias.com/current-affairs/mohammed-

ishtayeh/5cb2d23a1d5def05e1609acb/

Q85. Consider the following statements:

(1) In 1930, He launched Kalaram Temple movement.

(2) The Buddha and His Dhamma was the last book written by him.

(3) Independent Labour Party (ILP) in 1936 was formed under his leadership.

Above statements are about which of the following Personality?

A. Dr B. R. Ambedkar

Page 88: 11th May 2019 Live Test conducted by

B. Jyotirao Govindrao Phule

C. Jawaharlal Nehru

D. Madan Mohan Malaviya

Answer: A

Explanation:

Bhimrao Ramji Ambedkar is popularly known as Babasaheb, meaning ‘respected father’ in

Marathi and Hindi.In 1930, Ambedkar launched Kalaram Temple movement to enter Kalaram

Temple. The Reserve Bank of India (RBI) was based on the ideas that Ambedkar presented to

the Hilton Young Commission. The Poona Pact was an agreement made in 1932 between

Ambedkar and Mahatma Gandhi on the reservation of electoral seats for the depressed classes

in the legislature of British India government. He was independent India’s first law and justice

minister. He was the architect of the Constitution of India by serving as the Chairman of the

Constitution Drafting Committee. The Buddha and His Dhamma is the last book of Ambedkar

which is a scripture for those who follow Navayana Buddhism.Independent Labour Party (ILP) in

1936 was formed under his leadership. Hence, option A. is the correct answer.

Recently President Ram Nath Kovind paid homage to the architect of the Indian Constitution Dr

BR Ambedkar on his 128th birth anniversary.

Source: http://vajiramias.com/current-affairs/br-ambedkar-1891-

1956/5cb44f661d5def05e160d1d8/

Q86. With respect to “Nirbhay” missile, consider the following statements:

(1) It is a Long Range Super-Sonic Cruise Missile, which can be deployed from multiple

platforms.

(2) It has been indigenously designed & developed by Defence Research & Development

Organisation (DRDO).

Which of the statements given above is/are correct?

A. 1 only

B. 2 only

C. Both 1 and 2

D. Neither 1 nor 2

Page 89: 11th May 2019 Live Test conducted by

Answer: B

Explanation:

Statement (1) is incorrect: Nirbhay is a Long Range Sub-Sonic Cruise Missile, which can be

deployed from multiple platforms. It is capable of loitering and cruising at Mach 0.7 (sub-sonic)

at altitude as low as 100 meters. It can carry nuclear warhead.

Statement (2) is correct: It has been indigenously designed & developed by Defence Research &

Development Organization (DRDO).

Value Addition

Subsonic cruise missile flies at a speed lesser than that of sound. It travels at a speed of around

0.8 Mach. The well-known subsonic missile is the American Tomahawk cruise missile.

Supersonic cruise missile travels at a speed of around 2-3 Mach i.e.; it travels a kilometre

approximately in a second. The modular design of the missile and its capability of being

launched at different orientations enable it to be integrated with a wide spectrum of platforms

like warships, submarines, different types of aircraft, mobile autonomous launchers and silos.

BRAHMOS is the only known versatile supersonic cruise missile system which is in service.

Hypersonic cruise missile travels at a speed of more than 5 Mach. Many countries are working

to develop hypersonic cruise missiles. BrahMos Aerospace is also in the process of developing a

hypersonic cruise missile, BRAHMOS-II, which would fly at a speed greater than 5 Mach.

Source: http://vajiramias.com/current-affairs/nirbhay/5cb571571d5def05e5217c97/

Q87. The United Nations Educational, Scientific and Cultural Organization (UNESCO) seeks to

encourage the identification, protection and preservation of cultural and natural heritage

around the world considered to be of outstanding value to humanity.

In this context which of the following is/are the UNESCO World Heritage Sites?

(1) Notre-Dame Cathedral, Paris

(2) Churches and convents of Goa

(3) Victorian Gothic and Art Deco Ensembles of Mumbai

(4) Complexe du Capitole in Chandigarh

Select the correct answer using the code given below:

A. 3 only

Page 90: 11th May 2019 Live Test conducted by

B. 2 and 3 only

C. 2, 3 and 4 only

D. 1, 2, 3 and 4

Answer: D

Explanation:

Point (1) is correct: Notre-Dame de Paris meaning ‘Our Lady of Paris’ It is a medieval Catholic

cathedral located in Paris, France. The cathedral is consecrated to the Virgin Mary and

considered to be one of the finest examples of French Gothic architecture. The cathedral was

begun in 1160 under Bishop Maurice de Sully and was largely complete by 1260. The cathedral

was listed as part of the ‘Paris, Banks of the Seine’ UNESCO World Heritage Site in 1991. It is

the seat of the Archbishop of Paris; where Napoleon was crowned Emperor in 1804; and where

Joan of Arc was beatified in 1909. The landmark has been immortalised in Victor Hugo’s novel

Notre-Dame de Paris (in English, The Hunchback of Notre-Dame).

Point (2) is correct: The churches and convents of Goa, the former capital of the Portuguese

Indies – particularly the Church of Bom Jesus, which contains the tomb of St Francis-Xavier –

illustrate the evangelization of Asia. These monuments were influential in spreading forms of

Manueline, Mannerist and Baroque art in all the countries of Asia where missions were

established. It is a part of UNESCO World Heritage Site in 1986.

Point (3) is correct: The Victorian Gothic and Art Deco Ensembles of Mumbai is a collection of

19th century Victorian Neo Gothic public buildings and 20th century Art Deco buildings in the

Fort area of Mumbai in Maharashtra, India. Having become a global trading centre, the city of

Mumbai implemented an ambitious urban planning project in the second half of the

19th century. It led to the construction of ensembles of public buildings bordering the Oval

Maidan open space, first in the Victorian Neo-Gothic style and then, in the early 20th century,

in the Art Deco idiom. The Victorian ensemble includes Indian elements suited to the climate,

including balconies and verandas. It is a part of UNESCO World Heritage Site in 2018.

Point (4) is correct: The Architectural Work of Le Corbusier, an Outstanding Contribution to the

Modern Movement Chosen from the work of Le Corbusier, the 17 sites comprising this

transnational serial property is spread over seven countries and is a testimonial to the invention

of a new architectural language that made a break with the past. They were built over a period

of a half-century, in the course of what Le Corbusier described as “patient research”. The

Complexe du Capitole in Chandigarh (India), the National Museum of Western Art, Tokyo

(Japan), the House of Dr Curutchet in La Plata (Argentina) and the Unité d’habitation in

Page 91: 11th May 2019 Live Test conducted by

Marseille (France) reflect the solutions that the Modern Movement sought to apply during the

20th century to the challenges of inventing new architectural techniques to respond to the

needs of society. These masterpieces of creative genius also attest to the internationalization of

architectural practice across the planet. It is a part of UNESCO World Heritage Site in 2016.

Source: http://vajiramias.com/current-affairs/notre-dame-fire/5cb6dcbd1d5def0e0785a997/

Q88. Who among the following had compiled the Mahavira’s teachings as the Jain Agamas?

A. Kesi

B. Indrabhuti Gautama

C. Neminatha

D. Sudharma Swami

Answer: B

Explanation:

Mahavira’s teachings were compiled by Indrabhuti Gautama (his chief disciple) as the Jain

Agamas. Hence option B. is correct.

Mahavira is also known as Vardhamana. He was the 24 th tirthankara who revived Jainism. He

was born in 6th century BC into a royal Kshatriya family in present-day Bihar, India. He left

home at the age of 30 and became an ascetic. He practiced intense meditation for 12 years,

after which he attained Kevala Jnana (omniscience). After attaining Kevala Jnana, Mahavira

taught that observance of the five vows of ahimsa (non-violence), satya (truth), asteya (non-

stealing), brahmacharya (chastity), and aparigraha (non-attachment) is necessary for spiritual

liberation. He attained nirvana at the age of 72, and his body was cremated.

Source: http://vajiramias.com/current-affairs/mahavir-jayanti/5cb6c1e11d5def05e63ef356/

Q89. Which of the following pairs is/are correctly matched?

Page 92: 11th May 2019 Live Test conducted by

Select the correct answer using the code given below:

A. 3 only

B. 2 and 3 only

C. 1 and 3 only

D. 1, 2 and 3

Answer: D

Explanation:

According to the projections by experts during the tenth session of the United Nations Open-

ended Working Group on Ageing, the share of older persons (those aged 60 years or above) in

India’s population is projected to increase to nearly 20% in 2050. The Open-Ended Working

Group on Ageing was established by the UN General Assembly by resolution 65/182 on 21

December 2010.

Schemes/ Provisions made by the Government for the aged persons:

(i) Ministry of Social Justice and Empowerment: Under Rashtriya Vayoshri Yojana (RVY), aids

and assistive living devices are provided to senior citizens belonging to BPL category who suffer

from age related disabilities. The Scheme is being implemented by the “Artificial Limbs

Manufacturing Corporation (ALIMCO)”, a Public Sector Undertaking under this Ministry.

Page 93: 11th May 2019 Live Test conducted by

(ii) Ministry of Rural Development: It is providing Old age pension under the Indira Gandhi Old

Age Pension Scheme (IGNOAPS) which is a component of National Social Assistance Programme

(NSAP).

(iii) Ministry of Health and Family Welfare: It had launched the National Programme for Health

Care of the Elderly (NPHCE) during the 11th Plan period to provide health care facilities to the

elderly people through State Public health delivery system at primary, secondary and tertiary

levels, including outreach services.

(iv) Ministry of Finance: A number of incentives have been provided under the Income Tax Act,

1961, to a senior citizen. Atal Pension Yojana (APY) has been announced in 2015 wherein the

subscriber has been guaranteed a minimum pension on completion of 60 years of age based on

the periodic contribution being made by the subscriber. Hence, option D. is the correct answer.

Source: http://vajiramias.com/current-affairs/ageing/5cb83be91d5def05e9b79a6b/

Q90. Which of the following statement is not correct about Bacteriophage?

A. It is a type of bacteria that infects bacteria and archaea.

B. All bacteriophages are composed of a nucleic acid molecule that is surrounded by a protein

structure.

C. A bacteriophage attaches itself to a susceptible bacterium and destroys their host cell.

D. According to a recent study in the river Ganga, the bacteriophages were detected to be

approximately 3 times more in proportion than bacterial isolates.

Answer: A

Explanation:

According to the study, ‘Assessment of Water Quality and Sediment To Understand Special

Properties of River Ganga,’Ganga river water contains a significantly higher proportion of

organisms with antibacterial properties. The study was commissioned by the Union Water

Resources Ministry and was conducted by the Nagpur-based National Environmental

Engineering and Research Institute (NEERI), a CSIR lab. Other Indian rivers also contain these

organisms but the Ganga — particularly in its upper Himalayan stretches — has more of them,

the study suggests. In the river Ganga, the bacteriophages were detected to be approximately 3

times more in proportion than bacterial isolates.

A bacteriophage literally means ‘bacteria eater’ is a type of virus that destroys bacteria. It is a

virus that infects and replicates within bacteria and archaea. All bacteriophages are composed

Page 94: 11th May 2019 Live Test conducted by

of a nucleic acid molecule that is surrounded by a protein structure. A bacteriophage attaches

itself to a susceptible bacterium and destroys their host cell. Hence, option A. is the correct

answer.

Source: http://vajiramias.com/current-affairs/special-properties-of-river-

ganga/5cbacfc71d5def54a9127cea/

Q91. With reference to the Limited Liability Partnerships (LLPs) consider the following

statements:

(1) It is an alternative corporate business form that gives the benefits of limited liability of a

company and the flexibility of a partnership.

(2) The Union Finance Ministry implements the limited liability partnership Act of 2008.

Which of the statements given above is/are correct?

A. 1 only

B. 2 only

C. Both 1 and 2

D. Neither 1 nor 2

Answer: A

Explanation:

Statement (1) is correct: Limited Liability Partnerships (LLPs) is an alternative corporate

business form that gives the benefits of limited liability of a company and the flexibility of a

partnership. Hence LLP is called a hybrid between a company and a partnership. All limited

liability partnership is governed under the limited liability partnership Act of 2008.

Statement (2) is incorrect: The Corporate Affairs Ministry implements the limited liability

partnership Act of 2008.

Source: http://vajiramias.com/current-affairs/limited-liability-partnerships-

llps/5cbae4ea1d5def54ae29a8cd/

Q92. Which of the following countries is not included in a recently launched Asian Tea Alliance

(ATA)?

A. Indonesia

Page 95: 11th May 2019 Live Test conducted by

B. Sri Lanka

C. Japan

D. Thailand

Answer: D

Explanation:

The Asian Tea Alliance (ATA), a union of five tea-growing and consuming countries, was

launched in Guizhou in China.

Members of the alliance are Indian Tea Association, China Tea Marketing Association,

Indonesian Tea Marketing Association, Sri Lanka Tea Board and Japan Tea Association. Hence

option D. is correct.

Its mandate is to enhance tea trade, cultural exchanges, technology exchanges as well as

enhancing global consumption of tea, while creating a sustainability agenda for the future of

Asian tea. The forging of this alliance comes in the aftermath of the signing of a MoU in

December 2018 between the Indian Tea Association and China Tea Marketing Association to

promote green and black tea consumption in major tea markets of Europe, the U.S., Russia and

West Asia, besides India and China.

Source: http://vajiramias.com/current-affairs/asian-tea-alliance-

ata/5cbac0731d5def54abcd4f58/

Q93. With reference to the Lymph Node Metastasis, consider the following statements:

(1) Lymph fluid carries oxygen and other nutrients to the cells, and carries away waste products

like carbon dioxide (CO2) that flow out of the cells.

(2) The spread of cancer to a new part of the body is called metastasis.

Which of the statements given above is/are correct?

A. 1 only

B. 2 only

C. Both 1 and 2

D. Neither 1 nor 2

Answer: C

Page 96: 11th May 2019 Live Test conducted by

Explanation:

Statement (1) is correct: Our bodies have a network of lymph vessels and lymph nodes. But

instead of carrying blood, these vessels carry the clear watery fluid called lymph. Lymph fluid

carries oxygen and other nutrients to the cells, and carries away waste products like carbon

dioxide (CO2) that flow out of the cells. Lymph fluid also contains white blood cells, which help

fight infections.

Statement (2) is correct: Cancer can appear in the lymph nodes in 2 ways: it can either start

there or it can spread there from somewhere else. This spread of cancer to a new part of the

body is called metastasis.

A team of researchers from the National Institute of Biomedical Genomics, Kalyani, West

Bengal, has found Biomarkers for lymph node metastasis in oral cancer. It generally refers to a

measurable indicator of some biological state or condition. Biomarkers are often measured and

evaluated to examine normal biological processes, pathogenic processes, or pharmacologic

responses to a therapeutic intervention.

Source: http://vajiramias.com/current-affairs/biomarkers/5cbc48021d5def54b02bf37f/

Q94. With reference to the Fixed Maturity Plans (FMPs), consider the following statements:

(1) They are a class of debt funds that are open-ended.

(2) The tenure of an FMP can vary between a few months to a few years but it offers a

guaranteed return.

Which of the statements given above is/are correct?

Page 97: 11th May 2019 Live Test conducted by

A. 1 only

B. 2 only

C. Both 1 and 2

D. Neither 1 nor 2

Answer: D

Explanation:

Fixed Maturity Plan (FMP) is a fixed tenure mutual fund scheme that invests its corpus in debt

instruments maturing in line with the tenure of the scheme.

Statement (1) is incorrect: Fixed Maturity Plans (FMPs) are a class of debt funds that are close-

ended i.e. one can only invest in them at the time of a new fund offer and they come with a

specified maturity date, much like a fixed deposit (FD).

Statement (2) is incorrect: However FMPs don’t offer a guaranteed return but only pitch an

indicative yield that the investor then takes a bet on. The tenure of an FMP can vary between a

few months to a few years.

Investments in FMPs are more tax-efficient, since there are ‘indexation’ benefits linked to

capital gains, as opposed to tax on interest income in the case of an FD. FMPs, however, like

other debt funds come with interest rate risk and credit risk.

Source: http://vajiramias.com/current-affairs/fixed-maturity-plans-

fmps/5cbc2cc61d5def54abcd7495/

Q95. Arrange the following places from north to south:

(1) Colombo

(2) Batticaloa

(3) Minicoy

(4) Malé

Select the correct answer using the code given below:

A. 1-2-3-4

B. 2-3-1-4

Page 98: 11th May 2019 Live Test conducted by

C. 2-1-4-3

D. 3-2-1-4

Answer: D

Explanation:

The correct sequence is Minicoy- Batticaloa- Colombo- Male

Recently a series of blasts were carried out during Easter services in Churches in Negombo,

Batticaloa and Colombo in SriLanka.

Recently the Island Development Agency reviewed the progress made towards the program

“Holistic development of islands”. Three projects in Lakshadweep have been identified for issue

of bids. They include tourism projects in the islands of Kadmat, Minicoy and Suheli Cheriyakara.

Malé is the densely populated capital of the Maldives, an island nation in the Indian Ocean.

Recently Maldives President Ibrahim Mohamed Solih’s party won landslide victory in the recent

parliamentary elections.

Source: http://vajiramias.com/current-affairs/sri-lanka-easter-terror-

attack/5cbd84071d5def54b02c123c/

Q96. With which of the following organization is the World Earth Day celebrated worldwide on

22nd April?

A. UNESCO

Page 99: 11th May 2019 Live Test conducted by

B.World Wide Fund for Nature

C. UNFCCC

D. None of the above

Answer: D

Explanation:

Earth Day is observed every year on April 22. It was first celebrated in 1970. On April 22, 1970,

millions of people took to the streets to protest the negative impacts of 150 years of industrial

development. Earth Day Network (EDN) is the not for profit organisation that leads Earth Day

worldwide. Earth Day 2019 Theme is to Protect Our Species. Hence, option D. is the correct

answer.

Earth Day Network’s mission is to diversify, educate and activate the environmental movement

worldwide. Growing out of the first Earth Day, Earth Day Network is the world’s largest

recruiter to the environmental movement, working with more than 75,000 partners in nearly

192 countries to build environmental democracy.

Source: http://vajiramias.com/current-affairs/earth-day-2019/5cbd5d611d5def54ad349dab/

Q97. Significant Reduction Exceptions (SREs) from USA sanctions for India and seven other

countries for importing oil from which among the following countries?

A. China

B. Russia

C. Venezuela

D. Iran

Answer: D

Explanation:

U.S. Secretary of State recently announced that the U.S. will not renew exemptions from its

sanctions for importing oil from Iran. In November 2018, U.S. granted exemptions from its

sanctions for importing oil from Iran for a 180-day period for India and seven other countries

(namely China, Japan, South Korea, Turkey, Italy, Greece and Taiwan). These sanctions are also

known as Significant Reduction Exceptions (SREs) to existing importers of Iranian oil. Now, U.S.

has decided to not renew the exemptions from its sanctions for importing oil from Iran so as to

Page 100: 11th May 2019 Live Test conducted by

achieve its national security objectives and pressurize Iran on denuclearisation. Hence, option

D. is the correct answer.

Source: http://vajiramias.com/current-affairs/significant-reduction-exceptions-

sres/5cbe81041d5def54ad34bc44/

Q98. All disputes relating to the elections to Parliament and State Legislatures are under which

among the following jurisdictions of the Supreme Court?

A. Original Jurisdiction

B. Appellate Jurisdiction

C. Advisory Jurisdiction

D. A Court of Record

Answer: B

Explanation:

The authority to enact laws for elections to the offices of President and Vice-President and to

Parliament and State Legislatures has been reposed by the Constitution in Indian Parliament

(Articles 71 and 327). Laws relating to conduct of elections to municipalities, panchayats and

other local bodies are framed by the respective State Legislatures (Articles 243 K and 243 ZA).

All doubts and disputes relating to the elections to the office of President and Vice-President

are dealt with by the Supreme Court (Article 71), whereas the initial jurisdiction to deal with all

doubts and disputes relating to the elections to Parliament and State Legislatures vests in the

High Court of the State concerned, with a right of appeal to the Supreme Court (Article

329). The disputed matters relating to elections to municipalities, etc. are decided by the lower

courts in accordance with the laws made by the respective State Governments. Hence, option

B. is the correct answer.

Source: http://vajiramias.com/current-affairs/role-of-supreme-court-in-fair-

elections/5cbe78c71d5def54abcdaeee/

Q99. With reference to the Peatlands, consider the following statements:

(1) They occur only in the tropics.

(2) The world’s largest tropical peatland was discovered beneath the forests of the Congo Basin

in 2017.

Page 101: 11th May 2019 Live Test conducted by

Which of the statements given above is/are correct?

A. 1 only

B. 2 only

C. Both 1 and 2

D. Neither 1 nor 2

Answer: B

Explanation:

The recent United Nations Environment Assembly in Nairobi, Kenya, has adopted its first ever

resolution on peatlands, giving an impetus to conservation and restoration efforts.

Peatlands are a type of wetlands. It refers to the peat soil and the wetland habitat growing on

its surface. In some areas, year-round waterlogged conditions slow the process of plant

decomposition to such an extent that dead plants accumulate to form peat. Over millennia this

material builds up and becomes several metres thick.

Statement (1) is incorrect: Peatlands occur in almost every country on Earth, currently covering

3% of the global land surface.

Statement (2) is correct: In 2017, the world’s largest tropical peatland was discovered beneath

the forests of the Congo Basin in 2017.

Source: http://vajiramias.com/current-affairs/peatlands/5cc01f621d5def68f9271ee9/

Q100. Crimea in Eastern Europe lies on a peninsula stretching out from the south of Ukraine

between which of the following water bodies?

A. Black Sea and the Sea of Azov

B. Mediterranean sea and Black Sea

C. Tyrrhenian Sea and Adriatic Sea

D. Aegean Sea and Black Sea

Answer: A

Explanation:

Page 102: 11th May 2019 Live Test conducted by

Crimea, officially part of Ukraine, lies on a peninsula stretching out from the south of Ukraine

between the Black Sea and the Sea of Azov. It is separated from Russia to the east by the

narrow Kerch Strait. Hence, option A. is the correct answer.

In early 2014 Crimea became the focus of the worst East-West crisis since the Cold War, after

Ukraine's pro-Moscow president Viktor Yanukovych was driven from power by violent protests

in Kiev. Kremlin-backed forces seized control of the Crimean peninsula, and the territory, which

has a Russian-speaking majority, voted to join Russia in a referendum that Ukraine and the

West deem illegal. In the recently held election Ukrainians choose comedian Volodymyr

Zelensky as their next President.

Source: http://vajiramias.com/current-affairs/volodymyr-

zelensky/5cbec7781d5def68f63413f3/